You are on page 1of 62

BATCH - CLAT ACHIEVERS 2024

Mock Test [CLAT 2024] #06


[with Hints and Solutions]
DATE:15/07/2023

Duration: 120 Minutes Candidate Name: _______________


Max. Marks: 150 Admit Card No. : _______________

INSTRUCTIONS TO CANDIDATES

1. The question booklet (QB) contains 150 multiple-choice questions.


2. Write your Admit Card No. on the first page of the QB before starting the test.
3. Use the separate carbonized Optical Mark Reader (OMR) Response Sheet to
answer all the questions. Make sure to read the detailed instructions provided
with the OMR Response Sheet before starting the test.
4. If you notice any discrepancy in the QB, request the invigilator to replace both
the QB and OMR Response Sheet. Do not use the previous OMR Response
Sheet with the new QB.
5. During the test, write the QB No., OMR Response Sheet No., and sign in the
designated space/column on the Attendance Sheet provided.
6. Keep your Admit Card signed by the Invigilator, as it will be required during
the admissions process.
7. The QB for the Undergraduate Five-Year Integrated Programme is worth 150
marks. Each correct answer earns 1 mark, and each wrong answer results in a
deduction of 0.25 marks. There are no deductions for unanswered questions.
8. You can keep the QB and the candidate's copy of the OMR Response Sheet
after the test.
The use of any unfair means, including the possession of electronic devices such
as mobile phones, headphones, and digital watches, is strictly prohibited during
the test. Impersonation or any other fraudulent practice may be a criminal
offense and will lead to disqualification and possible legal action.

[1]
CONTENTS OF QUESTION PAPER

Subject Q. Nos. Page No.


English Language 1 – 25 3–8
Logical Reasoning 1 - 25 9 – 15
Current Affairs Including General Knowledge 1 - 30 16 – 20
Legal Reasoning 1 - 30 21 – 29
Quantitative Techniques 1 - 10 30
Answer Key 1-150 31
Hints & Solution 1-150 32 - 62

[2]
English Language
PASSAGE I AI in a portfolio carries risks and requires careful
Artificial Intelligence (AI) stocks should now be part consideration. Investors should seek professional
of most investors’ portfolios as Big Tech prepare for an advice before making any investment decisions. We
AI boom, suggests the CEO and founder of one of the believe that AI is the future, and we know from the past
world’s largest independent financial advisory, asset that early investors in innovative technologies often
management and fintech organisations. reap enormous rewards,” adds Green.
The comments from deVere Group’s Nigel Green 1. What is the viewpoint of Warren Buffett and Charlie
follow last week’s earnings reports from tech titans Munger regarding artificial intelligence (AI)?
including Microsoft, Alphabet (parent company of (A) They believe AI is overhyped and prefer
Google) Meta (parent company of Facebook, traditional intelligence.
Instagram and Whatsapp) and Amazon. (B) They expressed optimism about the potential of AI
It also comes after legendary investors Warren Buffett in the tech industry.
and Charlie Munger, the Chairman and CEO and Vice (C) They are skeptical about the financial performance
chairman respectively of Berkshire Hathaway, spent of tech giants in the AI sector.
hours this weekend at a shareholders’ meeting in which (D) They recommend including AI stocks in most
they expressed scepticism about AI. investors' portfolios.
“I am personally sceptical of some of the hype that is
going into artificial intelligence. I think old-fashioned
2. What was the focus of the Big Tech earnings reports
intelligence works pretty well,” Munger noted.
mentioned in the passage?
However, Nigel Green says: “Despite Buffett and
(A) The financial performance of tech giants in the AI
Munger’s scepticism, Big Tech last week posted robust
sector.
financial performance reports for first-quarter earnings.
The pace of growth has picked up noticeably. (B) The impact of AI on society and global business.
But the real story was the tech giants’ seemingly (C) The potential returns from investing in AI stocks.
relentless mania for AI. Most of corporate America (D) The guidance for managing an economic
clearly think that this is the future. The importance of downturn.
this cannot be overstated considering that just five tech
companies have made up two-thirds of the S&P 500’s 3. According to the passage, what was the dominant topic
gains so far this year. in the Big Tech earnings season?
Besides guidance, which suggests how companies are (A) Corporate guidance
positioned to manage an economic downturn, and data (B) Data on cost-cutting measures
on how effective cost-cutting measures have been, the (C) Robust financial performance reports
Big Tech earnings season was dominated by AI detail. (D) AI detail
The tech titans are fully aware of the enormous returns
that could be secured when AI starts to radically change 4. What is the potential impact of AI on society and global
the way businesses work and consumers live their lives. business, according to the passage?
“We fully expect that the volume of chat around AI will (A) It could lead to enormous rewards for early
ramp up in future earnings seasons,” says Green. investors
The AI chatbot ChatGPT became wildly popular in just (B) It could result in substantial losses for investors
a matter of weeks – and took off faster than social
(C) It could cause a decline in the tech industry
media platforms like TikTok or Instagram. Only two
(D) It could result in a decrease in employment
months after its launch in late November, it had 100
opportunities
million monthly active users in January, according to
reports.
It is because of the potential way that AI is expected to 5. What is the purpose of the AI chatbot ChatGPT
impact society and global business that Nigel Green mentioned in the passage?
now says that “AI stocks should have a place in most (A) To provide financial advice to investors
investors’ portfolios.” (B) To help businesses work more efficiently
The deVere CEO concludes: “We expect that (C) To serve as a social media platform
companies that have substantial AI interests are likely (D) To demonstrate the potential of AI
to benefit from the growth of the industry and this could
have potentially significant rewards for early investors.
But, of course, like any investment strategy, including

[3]
PASSAGE II
Two recent moves by the Telecom Regulatory (B) Revising the current centralised audit process to a
Authority of India(TRAI) have created some circle-wise approach.
discomfort for the industry. The first one relates to the (C) Eliminating the audit of billing and metering
audit of metering and billing of consumer services, practices for telcos.
where the regulator is examining whether a long-settled (D) Increasing the regulatory burden on telecom
practice needs to be re-looked. Whether subscribers operators.
have been billed correctly is audited by every telecom
operator, after which the same is examined by the 7. What concerns do telecom operators have regarding the
regulator as part of an external audit process. It is recent moves by the Telecom Regulatory Authority of
natural that some anomalies will get flagged and India (Trai)?
operators concerned would be directed to take (A) The potential increase in their regulatory burden
corrective measures. What is worrying the operators is due to micro-audits at the circle level.
a recent consultation paper by the regulator that seeks (B) The lack of centralization in the audit process of
to conduct micro-audits at the circle level, against the billing and metering.
current centralised process. Licences are awarded on a (C) The excessive cost of deploying new
circle basis and all other regulatory obligations are also technological mechanisms as demanded by the
computed on this basis. regulator.
In this context, it is fair for the Trai to feel that audit of (D) The possibility of facing penalties for past
billing and metering of telcos should also be done instances of predatory pricing and unlimited data
circle-wise. However, the telcos feel that they have offers.
centralised the process of audits and any change the
regulator is mulling would be superfluous and add to 8. How does the concept of "light touch regulation" align
their regulatory burden. The regulator is yet to come with the suggested approach for resolving issues
out with a final directive, but indications are that it may between telecom operators and the Telecom
not fully agree with the contention of the operators. If Regulatory Authority of India (Trai)?
it agrees, it is sure to demand the deployment of some (A) Light touch regulation allows for a more
new technological mechanisms, which may add to the centralized audit process, reducing the burden on
telcos’ cost. telecom operators.
The second issue relates to predatory pricing, which (B) Light touch regulation promotes the use of new
had arisen in 2017 post the launch of services by technological mechanisms to streamline billing
Reliance Jio and subsequently faded away. With the and metering audits.
launch of 5G services by Jio and Bharti Airtel rival (C) Light touch regulation favors conducting micro-
Vodafone Idea which is yet to start such services, has audits at the circle level to ensure regulatory
alleged that the two are indulging in predatory compliance.
pricing—that they are providing 5G at 4G rates and (D) Light touch regulation emphasizes open dialogue
offering unlimited data. While the regulator feels that and resolving issues through discussions among
the charge of predatory pricing is not correct as services all stakeholders.
are not being offered below cost, offering unlimited
data is against regulatory principle. 9. Which word best describes the approach favored by the
What could have got fixed by a mere change in Telecom Regulatory Authority of India (Trai) in
promotional language, has got muddied with the two resolving issues with telecom operators?
operators citing past instances of Vodafone Idea (A) Procrastination
indulging in predatory pricing and offering unlimited (B) Facilitation
data in some of its 4G packages. As a result, Trai will (C) Denunciation
now conduct a probe of all past offers by all the (D) Invalidation
operators to determine when the first violation took
place. It plans to pass a comprehensive order after this 10. Which of the following sentences from the passage
detailed examination is over. Rather than get into a contains a grammatical error?
regulatory overdrive over the issue of audit of billing (A) "Whether subscribers have been billed correctly is
and metering and unlimited offers, it would be better if audited by every telecom operator."
the Trai instead calls a meeting of all the operators and (B) "In this context, it is fair for the Trai to feel that
resolve such issues across the table. Light touch audit of billing and metering of telcos should also
regulation has become a hallmark of Trai; it should see be done circle-wise."
that this is maintained. (C) "What could have got fixed by a mere change in
promotional language, has got muddied with the
6. What is the Telcom Regulatory Authority of India two operators citing past instances."
(Trai) considering in relation to the audit of billing and (D) "The regulator is yet to come out with a final
metering of telcos? directive, but indications are that it may not fully
(A) Conducting micro-audits at the central level agree with the contention of the operators."
instead of the circle level.

[4]
PASSAGE III the mandate of the people. In a democracy, nothing less
A democratically elected government must be allowed will do.
to serve its constituents. This is the message at the heart
of the significant Supreme Court judgment this week 11. Which of the following weakens the argument for
that confirmed that the Aam Aadmi Party (AAP) allowing a democratically elected government to serve
government controlled bureaucrats assigned to its constituents, as presented in the passage?
departments that are legally within its purview. The (A) The Supreme Court judgment emphasizes the
105-page order by a Constitution bench headed by need for a cooperative federalism, which implies a
Chief Justice of India DY Chandrachud breathes life balanced power-sharing between elected
into the federal balance of power that is critical for representatives and bureaucratic officials.
effective management of India’s Capital, and upholds (B) The working relationship between the chief
the supremacy of elected representatives. The verdict minister's office and the lieutenant governor's
will hopefully put to an end an acrimonious turf battle office had been effective for several decades,
for control of power that has paralysed governance in demonstrating the viability of shared power.
Delhi for the past year and triggered unseemly political (C) The power struggle between the elected
squabbles. government and the lieutenant governor resulted
At the root of the dispute lies Delhi’s unique status as a in annulled appointments, policies in limbo, and
Union Territory with an elected assembly and breakdowns in communication, leading to
government, which also serves as the national Capital.
ineffective governance.
Since the first assembly elections in the city in 1993,
(D) The court's judgment highlights the importance of
the Capital has had two power centres: One in the chief
accountable and responsive civil service, which
minister’s office and the other at that of the lieutenant
suggests that elected representatives alone may
governor (LG) , a functionary who reports to the Union
not ensure efficient governance.
home ministry. For several decades, by serendipity or
arrangement, the two managed to work with each other,
12. Based on the information provided in the passage,
but this working relationship has frayed in recent years.
which of the following can be inferred about the
With mounting confrontation between Raj Bhavan and
relationship between the elected government and the
the chief minister’s office, the perception that the
lieutenant governor's office in Delhi?
elected government in the city was increasingly being
(A) The lieutenant governor's office has historically
rendered ineffectual took hold as the LG sought to
held more power and authority than the elected
expand his purview, especially after amendments to the
government in Delhi.
Government of National Capital Territory of Delhi Act
were passed in 2021. This stand-off created a vortex of (B) The power struggle between the elected
annulled appointments, notifications issued and then government and the lieutenant governor's office
withheld, policies caught in limbo, dharnas and street has negatively affected governance in Delhi.
protests and an apparent breakdown in communication (C) The recent Supreme Court judgment has resolved
between the two principals. Though the showdown the power struggle between the elected
began as an administrative affair, the political edge to government and the lieutenant governor's office.
the battle was unmistakable, especially because, on the (D) The amendments to the Government of National
ground, it often translated into a fight between the AAP Capital Territory of Delhi Act in 2021
and the Bharatiya Janata Party (BJP). The top court’s significantly expanded the purview of the elected
unanimous verdict will hopefully put to rest this government in Delhi.
unbecoming political fracas.
The court was clear that in a democratic form of 13. Based on the information provided in the passage,
government, the real power of administration must which of the following can be inferred about the
reside in the elected arm of the State and underlined relationship between the Aam Aadmi Party (AAP)
that an unaccountable and a non-responsive civil government and the Bharatiya Janata Party (BJP) in
service may pose a serious problem of governance. Delhi?
By stressing on the importance of cooperative (A) The Aam Aadmi Party (AAP) and the Bharatiya
federalism, the verdict cautions against the excesses of Janata Party (BJP) have collaborated effectively in
political overreach. Nevertheless, the judgment resolving the power struggle between the elected
showed the way forward – for responsible and sensitive government and the lieutenant governor's office.
governance, for politicians to focus on their (B) The power struggle between the Aam Aadmi Party
constituents rather than each other, and for respecting (AAP) government and the Bharatiya Janata Party

[5]
(BJP) has been the primary cause of ineffective Ironically, this break comes just five years after Mr
governance in Delhi. Khan was propelled to power by the army’s support
(C) The Bharatiya Janata Party (BJP) has supported and backroom machinations. While in power, Mr Khan
the Aam Aadmi Party (AAP) government in its often spoke of how the civilian government and the
efforts to assert control over assigned bureaucrats. army were on the same page, creating the potential for
(D) The Aam Aadmi Party (AAP) government and the tackling Pakistan’s problems. But the army became
Bharatiya Janata Party (BJP) have reached a wary of Mr Khan as he sought to interfere in the posting
mutual agreement to share power and of senior generals and secretly hobnobbed with some
responsibilities in governing Delhi. officers to retain his grip on power. Both the
parliamentary vote of confidence that ousted Mr Khan
14. Which of the following conclusions can be drawn from from power a year ago and the emergence of a rickety
the information provided in the passage? coalition government led by Shehbaz Sharif would not
(A) The power struggle between the Aam Aadmi Party
have happened without the go-ahead from the military.
(AAP) government and the Bharatiya Janata Party
(BJP) in Delhi has been successfully resolved Relations finally nosedived with Mr Khan’s repeated
through the intervention of the Supreme Court. public potshots at the military and repeated allegations
(B) The power struggle between the Aam Aadmi Party that the military wanted him dead.
(AAP) government and the Bharatiya Janata Party The army faces a stark choice. Most analysts believe
(BJP) has negatively impacted the effectiveness of the days when generals such as Zia-ul-Haq and Pervez
governance in Delhi. Musharraf removed civilian governments are over, and
(C) The power struggle between the Aam Aadmi Party
the military has shifted to a role where it can manage
(AAP) government and the Bharatiya Janata Party
(BJP) is a common phenomenon observed in the government by remote control. However, the
democratic governments worldwide. army’s deepening unpopularity and experience with Mr
(D) The power struggle between the Aam Aadmi Party Khan indicate that a hybrid regime is unworkable.
(AAP) government and the Bharatiya Janata Party Since the days of General Ashfaq Pervez Kayani, who
(BJP) has led to collaborative efforts in improving replaced Musharraf, several Pakistan Army chiefs have
governance in Delhi. spoken of steering clear in politics. But with its
interests and influence over Pakistan’s destiny at stake,
15. In the context of the passage, what does the term
"purview" most likely mean? it will be tough for the current chief, General Asim
(A) The legal authority or scope of control over a Munir to walk the talk on this promise. In a country
specific area or domain. wrecked by a floundering economy and escalating
(B) A state of disagreement or conflict between two terror strikes from the Tehreek-e-Taliban Pakistan, his
parties. next steps will be key.
(C) The act of intervening or mediating in a dispute or
conflict.
(D) The process of reaching a mutual agreement or 16. What is the main point emphasized in the passage?
understanding. (A) The arrest of former prime minister Imran Khan
has led to widespread protests against the
PASSAGE IV Pakistani military's interference in domestic
The Pakistani military’s history of repeatedly meddling politics, indicating a significant rift between
in domestic politics appears to have caught up with it, Khan's party and the military.
if the fallout of the arrest of former prime minister (PM) (B) Imran Khan's rise to power with the support of the
Imran Khan is anything to go by. Mr Khan was Pakistani military has resulted in strained relations
detained at a court complex in Islamabad with the help as Khan attempted to assert control over military
of a paramilitary force led by Pakistan Army officers, appointments and publicly criticized the military.
in connection with a corruption case. The dramatic (C) The Pakistani military, despite its shifting role in
operation unleashed protests that convulsed Pakistan as managing the government by remote control,
hundreds of Mr Khan’s supporters broke into the faces challenges in maintaining a hybrid regime
compound of the General Headquarters in Rawalpindi due to its growing unpopularity and conflicts with
and the corps commander’s residence in Lahore. These political figures like Imran Khan.
unprecedented scenes of anger against the military (D) General Asim Munir, the current chief of the
underlined that differences between Mr Khan’s Pakistani military, faces a crucial situation as he
Pakistan Tehreek-e-Insaf (PTI) and the de facto power attempts to balance the military's interests and
arbiter in the country have crossed the point of no influence over Pakistan's destiny amidst economic
return. challenges and escalating terror strikes.

[6]
17. What can be inferred from the passage regarding the 20. Which of the following best defines the term "potshots"
relationship between former prime minister Imran as used in the passage?
Khan and the Pakistani military? (A) Direct criticisms or attacks against the military
(A) Imran Khan had a harmonious relationship with (B) Casual and playful remarks aimed at diffusing
the Pakistani military throughout his tenure as tension
prime minister. (C) Secret negotiations and dealings to retain political
(B) Imran Khan's attempts to assert control over power
military appointments and criticize the military led (D) Intense and confrontational protests against the
to strained relations with the Pakistani military. government
(C) Imran Khan relied on the support of the Pakistani
military to gain power and maintain his position as PASSAGE V
prime minister. On 10th March 2023, China took the world by surprise
(D) Imran Khan's repeated public potshots at the when it announced that Saudi Arabia and Iran have
military and allegations against them created a agreed to resume diplomatic relations, in a peace deal
strong alliance between him and the Pakistani brokered by it. The peace deal offers hope for peace
military. and end to conflicts in the region. Among such conflicts
is the eight-year-long ongoing war in Yemen where
18. What can be inferred from the passage about the current Saudi-led coalition is pitched against Iran-backed
role of the Pakistani military in politics? Houthi rebels since March 2015. As per the peace deal,
(A) The Pakistani military has maintained a neutral both sides have agreed to end their support to the war
stance in politics since General Ashfaq Pervez in Yemen. It may be recalled that Saudi Arabia, along
Kayani's tenure. with regional Arab Allies and the US, launched
(B) The Pakistani military continues to actively airstrikes in Yemen on 26th March, 2015 in the air
intervene in politics, but with a more subtle assault codenamed "Operation Decisive Storm", in
approach. response to the Houthi’s armed conflict against the
(C) The Pakistani military's influence in politics has government of Yemen.
diminished, leading to a more stable government. Already, there are signs of the peace deal taking shape
(D) The Pakistani military is seeking to establish a in Yemen. On 09th April, Saudi and Omani delegations
hybrid regime by managing the government held talks in Sanaa with Houthi officials as well as the
remotely. head of Houthi Supreme Political Council, Mahdi al-
Mashat. The talks focused on an early reopening of
19. Which of the following weakens the argument that the Houthi-controlled ports and the Sanaa airport, nation
arrest of former prime minister Imran Khan signifies a rebuilding efforts and a clear timeline for foreign forces
significant shift in the relationship between the to exit the country. In the past few weeks, both sides
Pakistani military and elected representatives? have exchanged number of prisoners too, as a goodwill
(A) The protests triggered by the arrest of Imran Khan gesture. There were concerns that the armed conflict
were primarily driven by his supporters rather than could resume after Ramzan as the ceasefire in Yemen
being representative of a broader sentiment had expired in October 2022, but there is hope that the
against the Pakistani military. relative calm over past one year may finally translate
(B) The arrest of Imran Khan was carried out by a into cessation of hostilities.
paramilitary force led by Pakistan Army officers, The conflict in Yemen however, goes well beyond the
indicating the military's direct involvement in the war since March 2015. The seeds of the unrest and
case. conflict have a long history, mostly internal to Yemen
(C) Imran Khan's repeated public potshots at the and a long-lasting peace can only prevail only once
military and allegations against them have both external and internal conflicts are resolved.
previously strained his relationship with the Internal Conflict
Pakistani military. The roots to the internal conflict in Yemen go back to
(D) The parliamentary vote of confidence that ousted 22nd May 1990 when the erstwhile two Yemeni
Imran Khan from power a year ago was facilitated nations were united and brought under a single flag.
by the military's go-ahead, suggesting an ongoing People’s Democratic Republic of Yemen (South
collaboration between the military and elected Yemen) merged with Arab Republic of Yemen (North
representatives. Yemen) to form a new country, ‘Republic of Yemen’.
It was a forced union as, Yemen, since the 19th century,

[7]
had been divided in two separate entities along a north- 23. What is the primary theme of the above passage?
south divide. (A) China's role in international diplomacy.
North Yemen was part of the former Ottoman Empire, (B) The political history of Yemen and the peace deal
while South Yemen fell under British influence in 1839
between Saudi Arabia and Iran.
when the British captured the port of Aden. After the
(C) The internal and external conflicts in Yemen, their
collapse of the Ottoman Empire in 1918, North Yemen
became an independent republic while South Yemen historical roots, and the recent peace deal that
continued under the British until Britain withdrew in offers hope for resolution.
1967, after which it was largely under control of (D) The operation of the Houthi Supreme Political
communist forces, till unification in 1990. Council in Yemen.

21. What historical event triggered the internal conflict in 24. Based on the passage, what term best describes the
Yemen? historical relationship between North Yemen and South
(A) The invasion of Saudi Arabia. Yemen before their unification in 1990?
(B) The unification of North and South Yemen in (A) Cooperation
1990. (B) Division
(C) The airstrikes launched by Saudi Arabia and the (C) Symbiosis
US in 2015. (D) Collaboration
(D) The British withdrawal from South Yemen in
1967. 25. What undermines the hope for a lasting peace in
Yemen, despite the recent peace deal between Saudi
22. What role does the peace deal between Saudi Arabia Arabia and Iran?
and Iran play in the context of the war in Yemen? (A) The historical conflict between North and South
(A) It allows for China's military intervention in Yemen.
Yemen. (B) The lack of involvement of the United States in the
(B) It leads to the end of support for the war in Yemen peace process.
from both sides. (C) The ongoing support of external powers to the
(C) It imposes sanctions on the Houthi rebels. warring factions.
(D) It provides for the deployment of UN (D) The reluctance of Houthi rebels to participate in
peacekeeping forces in Yemen. peace talks.

[8]
Logical Reasoning
PASSAGE I 1. What is the main idea conveyed by the passage?
In the past four decades and more, the structure of (A) India's economic structure has undergone
India’s economy has changed quite dramatically. significant changes over the past four decades,
Compared to 1980-81, using current prices, the share with the services sector becoming dominant.
of “agriculture and allied activities” has dropped (B) India's economic growth has accelerated due to
from 38 per cent of GDP to 21 per cent, while that of the dominance of the rapidly growing services
services has grown from 37 per cent to 53 per cent. sector.
That of industry (including construction and utilities) (C) The shift in weighting among sectors has
has remained more or less unchanged at 26 per cent. resulted in India's potential for annual growth
So agriculture, the slowest-growing segment of the reaching at least 7 percent.
economy, has shrunk relative to the others, while the (D) The International Monetary Fund (IMF) has
services sector (the fastest-growing) has become the expressed concerns about India's growth
dominant one. What does this structural change mean potential due to the impact of the pandemic and
for overall economic growth? Assuming the sector- other factors.
wise growth rates remain unchanged, the dominance
acquired by the rapidly growing services sector 2. To what degree is the statement about India's
means that economic growth as a whole should have potential for growth supported by the information
accelerated. provided in the passage?
Given the shifts in weighting among the three sectors, (A) The statement is highly supported, as the
what was 5.5 per cent average growth in the 1980s passage presents various factors that suggest
should translate into at least 6.3 per cent growth now. India's potential growth rate should be at least 7
Now factor in other changes, like life expectancy. percent.
This was 54 years in 1980, and is currently estimated (B) The statement is moderately supported, as the
at 70 years. In other words, the average Indian no passage mentions some factors that could
longer dies while still in their working age. That contribute to India's potential growth but does
should have improved productivity, as should the not provide comprehensive evidence.
rapid spread of education, including post-school (C) The statement is minimally supported, as the
education, where enrolment levels have grown passage discusses changes in India's economy
sharply — although educational quality is an issue. but does not directly address its potential growth
Compound these with the increased rate of rate.
investment in fixed capital (up from 19.7 per cent of (D) The statement is unsupported, as the passage
GDP in 1980-81 to 28.6 per cent before the primarily focuses on the shift in India's
pandemic), and efficiency boosters like the spread of economic structure and the IMF's assessment of
digitisation. its growth rate.
Some of these would have contributed to the sectoral
shift. Adjusting for that, one could reasonably argue 3. What can be inferred from the information provided
that the Indian economy’s potential for annual growth in the passage?
should have become at least 7 per cent — the (A) The IMF's assessment of India's growth rate is
threshold beyond which a country is assumed to have widely accepted by the government and
achieved rapid growth. Indeed, in the two decades domestic commentators.
prior to the pandemic, India through many ups and (B) India's rapid spread of education and digitization
downs averaged annual growth not far short of that has positively impacted productivity and
magic number. So it is striking (though perhaps not economic growth.
surprising) that the International Monetary Fund (C) The average life expectancy of Indians has
(IMF) thinks India’s potential for growth has increased significantly, leading to higher
suffered, partly on account of the pandemic but also productivity and economic growt
for other reasons. Indeed, Pierre-Olivier Gourinchas, (D) The pandemic has had a significant negative
the IMF chief economist, while commenting last impact on India's potential for economic growth.
month on global growth prospects, contended that
India’s growth in the current year (estimated by the 4. Which option would strengthen the argument that
IMF at 5.9 per cent) was near the limit of its capacity, India's potential for annual growth should be at least
and there was no “output gap” between actual and 7 percent?
potential growth. Mr Gourinchas’s word is not (A) The services sector has consistently
gospel; indeed, the growth assessments of the outperformed other sectors in terms of growth
government and the Reserve Bank this year are rate over the past four decades.
higher. But domestic commentators need to engage (B) The rate of investment in fixed capital has
seriously with the issue of the rate of growth at which remained stable, indicating a favorable
India will have no “output gap”. environment for economic growth.

[9]
(C) India's educational quality has improved of cyberbullying of women, young or old, are on the
significantly, leading to a highly skilled rise too. The absence of a support structure that can
workforce and increased productivity. help them access timely counselling compounds the
(D) The government and the Reserve Bank of India
challenge. The fifth National Family Health Survey
have projected higher growth rates for the
current year compared to the IMF's estimate. revealed that a majority in the 15-49 years age group
had never used a smartphone. However, this may
5. Which option would weaken the argument that change soon: the 2022 McAfee study found that
India's potential for annual growth should be at least smartphone usage among Indian children is as high
7 percent? as 83% — 7% more than the global average. A
(A) The IMF's assessment of India's growth rate protective edifice must be in place at the earliest.
aligns with the growth assessments of the
There needs to be some form of gatekeeping when it
government and the Reserve Bank.
(B) India's economy has faced significant comes to permitting children to use smartphones.
challenges, such as income inequality and a lack Parents and teachers can play a constructive role here,
of inclusive growth. but the thin line separating regulation and
(C) The services sector, despite its dominance, has surveillance must be adhered to. Women must also be
shown signs of slowdown and reduced growth sensitised to insulate themselves from harmful
potential. content. Prevention is better than cure.
(D) The pandemic has had a minimal impact on
India's potential for economic growth, allowing
for a rapid recovery. 6. Which of the following weakens the argument that
heavy smartphone use leads to addiction and causes
PASSAGE II serious health issues?
Technological marvels, such as the smartphone, have (A) Studies have shown that moderate smartphone
made modern life convenient. But convenience can use has no negative impact on mental health.
come at a price. There is evidence to indicate that the (B) Smartphone addiction is not a widespread issue
heavy use of smartphones leads to addiction, which and only affects a small percentage of the
is causing serious health issues among adults and population.
children. Some of the manifestations of impaired (C) The use of smartphones can actually improve
health on account of smartphone addiction include cognitive function and self-esteem in
fatigue, poor cognitive function, low self-esteem and individuals.
so on. But there is a gendered angle to this (D) The health issues attributed to smartphone
vulnerability. A recent report by Sapien Labs, a addiction can be easily mitigated through regular
Washington-based non-profit, has suggested that the exercise and healthy lifestyle choices.
deleterious effects of smartphones are more
pronounced on the mental health of women. Age of 7. Which of the following statements must be false
First Smartphone/Tablet and Mental Well-being based on the information provided?
Outcomes, which gathered data from 27,969 (A) The age at which a person gets their first
respondents aged between 18-24 years across 41 smartphone has no impact on their mental well-
countries, connected the parameters associated with being.
mental well-being to the age of first ownership of (B) Men are more likely to experience suicidal
smartphones. According to the report, those who got thoughts and hallucinations as a result of
their first smartphones at the age of six were found to smartphone addiction.
be experiencing suicidal thoughts and hallucinations; (C) The mental health effects of smartphone
they also demonstrated aggressive behaviour. But the addiction are more pronounced in women
number of women affected — 74% — was much compared to men.
higher than the number of men — 42%. These figures (D) Cyberbullying of women is a rising concern
progressively declined — more among men than associated with smartphone use.
women — as the age of ownership increased.
The difference in impact on the genders could be 8. What can be inferred from the passage?
viewed through the prism of structural inequities. (A) Women are more susceptible to smartphone
Smartphones disseminate content coated with addiction compared to men.
misogyny and gender discrimination. As consumers, (B) The use of smartphones at an early age is directly
women users are thus at a greater risk of suffering a linked to aggressive behavior.
range of debilitating mental conditions, such as (C) The content on smartphones often perpetuates
inclination towards self-injury and depression. Cases misogyny and gender discrimination.
[10]
(D) The prevalence of smartphone usage among president’s. The sudden change of portfolio, with
Indian children is higher than the global average. little preparation and no explanation, therefore, has
prompted speculation that the government may be
9. What is a flaw in the argument presented in the looking for a less abrasive relationship with the
passage? Supreme Court at the present time. The imminence of
(A) The passage fails to provide any evidence or the Lok Sabha elections could be one reason to
research studies to support the claim of smooth matters with the higher judiciary. Mr Rijiju’s
smartphones causing addiction and health replacement, Arjun Ram Meghwal, made it clear that
issues. there would be no confrontation with the courts, and
(B) The argument does not consider other factors the relationship between the executive and the
that could contribute to mental health issues, judiciary would remain cordial, constitutional and
such as underlying mental health conditions or within the laid-down boundaries.
socioeconomic factors. Since it is difficult to believe that either Mr Rijiju or
(C) The passage relies solely on data from a single Mr Meghwal would exceed his brief, the latter’s
report by Sapien Labs without considering other statement indicates a desire to change the impression
studies or perspectives on the topic. the government has so far given regarding the
(D) The argument assumes that regulating and judiciary. Mr Rijiju’s aggression made no dent in the
restricting smartphone use among children and judiciary’s stand, neither did it ruffle the dignity of its
sensitizing women to harmful content will seniormost members; hence that route is no longer
effectively prevent mental health issues. viable. At least, not for the moment. Removing the
minister tasked with scathing remarks and giving him
10. What is the most appropriate course of action based a comparatively less important ministry may change
on the information provided? public perception and suggest the government’s shift
(A) Implement strict regulations and surveillance in attitude. Mr Modi and his men seem to have an
measures to monitor smartphone usage among unshakeable faith in optics. The situation also offered
children. the scope to bestow on Mr Meghwal, a Dalit from
(B) Conduct further research to determine the exact Rajasthan and minister of state for culture and
impact of smartphone use on mental health in parliamentary affairs, independent charge of the law
different demographic groups. ministry before elections in Congress-ruled
(C) Create a support structure to provide timely Rajasthan. It is a rare responsibility for a junior
counseling and assistance to individuals minister, especially since the government does not
experiencing mental health issues related to lack suitable candidates. Perhaps the high command
smartphone addiction. is making the best of a not-too-successful job.
(D) Educate parents and teachers on the importance
of gatekeeping and responsible smartphone use 11. Which of the following, if true, would most weaken
among children. the speculation about the government's motive
behind Kiren Rijiju's change in portfolio?
PASSAGE III (A) Kiren Rijiju voluntarily requested a transfer to
Powerful reasons alone can cause a change in the
the earth sciences ministry due to his personal
portfolio of a high-profile minister. Kiren Rijiju, the
former Union law minister, has now been given the interest in the subject.
task of looking after earth sciences instead. Mr Rijiju (B) The Supreme Court recently praised Kiren Rijiju
has been best known for his unflagging criticism of for his efforts in advocating for judicial reforms.
the collegium system of judges’ appointments; (C) The government has maintained a cordial
according to him, not senior judges but the relationship with the Supreme Court throughout
government should be appointing and transferring Narendra Modi's tenure as Prime Minister.
judges. That Mr Rijiju was representing the Narendra
(D) The Lok Sabha elections have been postponed,
Modi-led government’s opinion was seldom in doubt.
eliminating the need for the government to
The micro-managing propensities of the Bharatiya
Janata Party’s high command also suggested that his establish a smoother relationship with the
increasingly confrontational rhetoric — calling some judiciary at present.
unnamed retired judges part of the ‘anti-India gang’,
for instance — probably had his superiors’ approval.
After all, his remarks ran parallel to the vice-
[11]
12. Which of the following must be true based on the 15. What should the government do to address the
information provided? situation described in the passage?
(A) Kiren Rijiju's criticism of the collegium system (A) Take immediate steps to repair Kiren Rijiju's
of judges' appointments had no impact on the relationship with the judiciary and reinstate him
judiciary's stance. as the Union law minister.
(B) Arjun Ram Meghwal's appointment as the new (B) Clearly explain the reasons behind Kiren Rijiju's
law minister was primarily influenced by his change in portfolio to the public and ensure
seniority within the Bharatiya Janata Party. transparency in the decision-making process.
(C) Kiren Rijiju's confrontational rhetoric towards (C) Appoint a new high-profile minister who can
retired judges was openly supported by the vice- continue Kiren Rijiju's aggressive approach
president. towards the collegium system of judges'
(D) Kiren Rijiju's transfer to the earth sciences appointments.
ministry was a strategic move by the (D) Postpone the Lok Sabha elections to allow for a
government to strengthen its relationship with longer period of smooth relations with the
the Supreme Court. Supreme Court.
13. Based on the passage, which of the following can be
inferred about Kiren Rijiju's aggressive approach PASSAGE IV
towards the judiciary? In the 1870s, German physician Robert Koch was
trying to curtail an epidemic of anthrax that was
(A) Kiren Rijiju's aggressive remarks were sweeping local farm animals. Other scientists had
welcomed by the senior judges of the Supreme seen a bacterium, Bacillus anthracis, in the victims’
Court. tis-sues. Koch injected this microbe into a mouse –
which died. He recovered it from the dead rodent and
(B) Kiren Rijiju's confrontational rhetoric did not
injected it into another one – which also died.
align with the government's opinion on judges' Doggedly, he repeated this grim process for over 20
appointments. generations and the same thing happened every time.
(C) The government's high command disapproved Koch had unequivocally shown that Bacillus
anthracis caused anthrax. This experiment, and those
of Kiren Rijiju's increasingly aggressive stance. of contemporaries like Louis Pasteur, confirmed that
(D) Kiren Rijiju's aggressive approach towards the many diseases are caused by microscopic organisms.
judiciary negatively affected the government's Microbes, which had been largely neglected for a
couple of centuries, were quickly cast as avatars of
image. death. They were germs, pathogens, bringers of
pestilence. Within two decades of Koch’s work on
14. Which of the following best identifies a flaw in the anthrax, he and many others had discovered that
bacteria were also associated with leprosy,
author's reasoning in the passage?
gonorrhoea, typhoid, tuberculosis, cholera,
(A) The author assumes that Kiren Rijiju's diphtheria, tetanus, and plague. Microbes became
aggressive remarks were the primary reason for synonymous with squalor and sickness. They became
his transfer to the earth sciences ministry. foes for us to annihilate and repel. Today, we know
this view is wrong – as I explain in my new book I
(B) The author assumes that the government's Contain Multitudes. Sure, some bacteria can cause
motive behind Kiren Rijiju's change in portfolio disease, but they are in the minority. Most are
is solely related to its relationship with the harmless, and many are even beneficial. We now
know that the trillions of microbes that share our
Supreme Court. bodies – the so-called microbiome – are an essential
(C) The author assumes that the government's part of our lives. Far from making us sick, they can
decision to give Arjun Ram Meghwal protect us from disease; they also help digest our
food, train our immune system, and perhaps even
independent charge of the law ministry is solely
influence our behaviour. These discoveries have
based on optics. shifted the narrative. Many people now see microbes
(D) The author assumes that Kiren Rijiju's transfer as allies to be protected. Magazines regularly warn
to a less important ministry will significantly that antibiotics and sanitisers might be harming our
health by destroying our microscopic support system.
change public perception of the government's Slowly, the view that ‘all bacteria must be killed’ is
attitude towards the judiciary. giving ground to ‘bacteria are our friends and want to
help us’.
[12]
The problem is that the latter view is just as wrong as 18. Based on the information in the passage, which of the
the former. We cannot simply assume that a following can be inferred?
particular microbe is ‘good’ just because it lives
inside us. There’s really no such thing as a ‘good (A) The majority of bacteria found in the human
microbe’ or a ‘bad microbe’. These broad-brush body are harmful and can cause diseases.
terms belong in children’s stories. They are ill-suited (B) The perception of bacteria as exclusively
for describing the messy, fractious, contextual
harmful has remained unchanged since Robert
relationships of the natural world. In reality, bacteria
exist along a continuum of lifestyles. If they do us Koch's experiments in the 1870s.
harm, we describe them as parasites or pathogens. If (C) The variability in the behavior of bacteria
they exist neutrally, we call them commensals. If they suggests that their impact on the host is
benefit us, we bill them as mutualists. But these are
hardly fixed categories. Some microbes can slide influenced by factors such as strain and host
from one end of this parasite-mutualist spectrum to species.
the other, depending on the strain and on the host they (D) The discovery of the human microbiome has led
find themselves in. For example, the Wolbachia
bacteria infect some 40 percent of insects; in some to the development of effective treatments for
species, these microbes are sexual parasites that kill diseases caused by harmful bacteria.
or manipulate males, whereas in others, they behave
as living dietary supplements that provide vitamins
19. Which of the following statements, if true, would
missing from the host’s diet.
strengthen the argument presented in the passage?
16. Which of the following best summarizes the main (A) Recent research has demonstrated that the
idea of the passage? elimination of certain strains of bacteria from the
(A) Microbes were initially viewed as agents of human microbiome can lead to an increased risk
disease and destruction, but recent discoveries
of autoimmune diseases.
have revealed their essential role in human
(B) Scientists have identified a specific strain of
health and well-being.
(B) Robert Koch's experiments in the 1870s bacteria that is universally beneficial to all
revolutionized the understanding of bacteria and human hosts.
their relationship to diseases. (C) The use of antibiotics has significantly reduced
(C) The view that all bacteria must be eradicated is the prevalence of harmful bacteria in human
gradually being replaced by the recognition of populations.
their beneficial functions in the human
(D) Bacterial infections are responsible for the
microbiome.
majority of deaths worldwide, highlighting the
(D) The categorization of bacteria as "good" or
"bad" is overly simplistic and fails to capture the urgent need for eradication strategies.
complex and variable nature of their interactions
with hosts. 20. Which of the following statements can be concluded
based on the information in the passage?
17. Which of the following assumptions is made in the (A) The discovery of the human microbiome has
passage? completely transformed the understanding of
(A) The study of bacteria in the 19th century was
bacteria and their interactions with hosts.
largely responsible for the initial negative
(B) Bacteria that are beneficial in one host species
perception of microbes as harmful entities.
(B) The discovery of the human microbiome has will always exhibit the same beneficial behavior
completely overturned the understanding of in other host species.
microbes as disease-causing agents. (C) The categorization of bacteria as parasites,
(C) The categorization of bacteria as parasites, commensals, or mutualists is a flawed
commensals, or mutualists is a valid and oversimplification of their complex nature.
accurate representation of their interactions with (D) The negative perception of bacteria as harmful
hosts.
entities has remained unchanged since the 19th
(D) The recognition of the beneficial role of
century.
microbes in human health has led to a complete
disregard for the potential harm caused by
certain bacteria.

[13]
PASSAGE V It is also important to recognize that reducing
Prejudice is a serious problem in society that can have prejudice is not a one-time fix, but an ongoing
detrimental effects on individuals and the broader process that requires sustained effort and
community. It often stems from a lack of personal commitment.
experience with individuals from a particular social Ultimately, reducing prejudice requires a collective
group, combined with media portrayals and the effort from individuals, organizations, and
beliefs of family and peers. Prejudice can manifest in communities. By working together to challenge bias
negative attitudes towards groups based on race, sex, and foster empathy, we can create a more equitable
gender identity, disability, obesity, and other and inclusive society where everyone has the
dimensions. opportunity to thrive. It is our responsibility to
Prejudice can appear in various contexts, such as actively address prejudice and discrimination in all its
workplaces, schools, and shops, and can be severely forms and to build a better world for ourselves and
damaging when it carries over into consequential future generations.
situations like employment and healthcare access.
Studies have shown that systemic discrimination 21. According to the passage, what is the potential harm
affects Black people, women, and people with of prejudice?
disabilities more than others, leading to poverty and (A) It can lead to anxiety, depression, and other
other negative outcomes. Furthermore, experiencing psychological disorders
prejudice can have serious mental health costs, (B) It can cause absenteeism and decreased
including anxiety, depression, and PTSD. This, in productivity
turn, can lead to absenteeism and decreased (C) It can lead to poverty and lack of access to
productivity. healthcare.
Fortunately, there are interventions that have been (D) It can reinforce existing prejudices in society.
proven to reduce prejudice, including cognitive
retraining and perspective-taking. Cognitive 22. The passage suggests that cognitive retraining can be
retraining involves exposing people to counter- an effective intervention for reducing prejudice. What
stereotypical information to challenge their biases, is the assumption underlying this suggestion?
while perspective-taking fosters empathy by (A) People can be persuaded to change their biases
encouraging people to consider the experiences of through exposure to counter-stereotypical
members of other social groups. These interventions information
have been tested in various settings, including (B) The prevalence of prejudice in society is too
primary and secondary schools, as well as doctors' great to overcome
offices, with positive results. (C) Prejudice is primarily caused by a lack of
Contact theory, which involves encouraging and personal experience with other social groups
facilitating contact between people from different (D) The negative effects of prejudice can be reversed
social groups, has been a popular approach to through psychological interventions
reducing prejudice, but emerging findings suggest
that it may not be the most effective method. The 23. The passage suggests that contact theory may not be
quality and nature of the contact matter, and forced or the most effective way to reduce prejudice. Which of
insincere contact may actually reinforce existing the following, if true, would strengthen this
prejudices. Instead, interventions like cognitive suggestion?
retraining and perspective-taking may be more (A) Contact that is perceived as forced or insincere
effective in reducing prejudice. may actually reinforce existing prejudices
In conclusion, prejudice is a complex issue with far- (B) The quality and nature of the contact between
reaching implications for individuals and society as a people from different social groups matters as
whole. While prejudice-reduction interventions such well
as cognitive retraining and perspective-taking have (C) Studies have shown that contact between people
shown promising results, more work needs to be done from different social groups can reduce
to fully understand the best ways to combat prejudice prejudice
effectively. This includes ongoing research into the (D) Contact theory has been a popular approach to
causes and consequences of prejudice, as well as the tackling prejudice in society
development and testing of innovative intervention
strategies.

[14]
24. Which of the following can be inferred from the 25. Based on the passage, which of the following is a
passage about the mental health costs of prejudice? potential limitation of perspective-taking as an
(A) Prejudice can lead to absenteeism, which in turn intervention for reducing prejudice?
can lead to decreased productivity (A) Perspective-taking can foster empathy and
(B) Prejudice can lead to anxiety, depression, and reduce prejudice
other psychological disorders (B) Perspective-taking involves encouraging people
(C) Prejudice primarily affects people who to consider the experiences and perspectives of
experience poverty and lack of access to members of other social groups
healthcare (C) Perspective-taking may not be effective for
people who already hold deeply ingrained biases
(D) Prejudice can be severely detrimental to (D) Perspective-taking has been proven to work in
individuals and broader society the real world, including and beyond contact

[15]
Current Affairs, including General Knowledge
PASSAGE I Cherpulassery mural remains a class apart in both
The Wall of Peace, a great work of modern mural art magnitude and subject.
on the 700-feet long compound wall of the
Government Vocational Higher Secondary School at 1. Which of the following is not true about Mural
Cherpulassery, was inaugurated on Monday. Paintings?
Minister for Local Administration M.B. Rajesh (A) It is made on the walls of caves and palaces.
dedicated the wall to the nation more than three and
(B) True fresco paintings are created when the
a half years after it was completed by Banaras Hindu
surface of the wall is dry, allowing the colors to
University’s assistant professor (X) and his students.
The mural, in dimmed golden shade done with penetrate deeply.
cement and sand on the 10-ft-high wall, has already (C) The earliest evidence of murals can be found on
won many an accolade, including the Eurasia World the walls of Ajanta and the Bagh caves.
Record for the largest public art project. No one can (D) It captures a moment in time by showcasing the
pass this century-old school wall without noticing activities of a particular civilization's
his work of art. He converted the idea of the Wall of inhabitants.
Peace that had dawned upon him over a decade ago
2. Which of the following will replace (X) in the
when the school authorities approached him for his
following passage?
creative contribution. The word ‘peace’ has been
embossed on the wall in 250 world languages. Art (A) Antim Pareta
lovers from countries like Austria, Israel, Hungary, (B) Srinivasa Babu Angara
and the U.S. who visited Cherpulassery and (C) Suresh K. Nair
neighbouring villages stood in awe in front of the (D) Geetika Singh
wall as they read ‘peace’ in their respective
languages. Apart from the word ‘peace’, He used the 3. Only two examples of cave paintings from the Gupta
struggle for freedom as the key motif of his biggest
era in ancient India are now known. Ajanta cave
mural done under the banner of the school
paintings are one of these. Where is the other
development committee named School Project
Aiming Centre of Excellence (SPACE). “It is meant surviving example of Gupta paintings?
to promote freedom and peace among the peace,” he (A) Nasik caves
said, sitting beneath one of the 29 giant banyan trees (B) Ellora caves
that formed a majestic canopy over the wall. He (C) Bagh caves
portrayed the moon at the start and the sun at the end (D) Lomas Rishi caves
symbolically showing humanity’s transition from
darkness to light, and from bondage to freedom and
4. The “Bhimbetka rock shelters” in Madhya Pradesh
peace. The wall has also been a tribute to
Cherpulassery’s local heroes of freedom struggle. are renowned for their prehistoric rock paintings.
“As a student, I was influenced by the book Khilafat Which period do these paintings primarily belong
Smaranakal penned by freedom fighter to?
Moozhikkunnathu Brahmadathan Namboodiripad. It (A) Palaeolithic period
describes the cruel treatment meted out to him by the (B) Neolithic period
British, who tied him to a horse and dragged him to (C) Mesolithic period
prison. I could give life to those disturbing images
(D) Chalcolithic period
on the wall along with several others,” said Mr
Suresh. Cherpulassery’s traditional cultural
activities like Kaalavela, and Karalmanna’s famous 5. Who is the current Minister of Tourism & Culture?
Pandibhojanam too have found a place on the wall. (A) Parshottam Rupala
Mahatma Gandhi’s visit to the region is also on the (B) Piyush Goyal
Wall of Peace. The Wall of Peace dwarfed another (C) G. Kishan Reddy
spectacular mural Mr Suresh did at India-Pakistan (A) Meenakshi Lekhi
Wagah Border back in 2014. The 200-feet long
cement mural at Wagah is about Partition. But the

[16]
PASSAGE II 6. Which of the following statements is/are correct as
Cricketer Rishabh Pant’s accident near Roorkee, per the report?
resulting in some injuries, has once again drawn 1. Driving under the influence of drugs/alcohol
attention to the problem of road safety in India. Nitin contributed to 1.9% of deaths from accidents.
Gadkari, Minister of Road Transport and Highways,
2. Nearly 90% of deaths on the road were due to
Government of India, recently said that the Indian
speeding, overtaking, and dangerous driving.
road accident scenario, with 415 deaths and many
injured every day, is more serious than Covid-19. (A) Only 1
This is a frank admission that even with (B) Only 2
comprehensive road safety programmes, India’s (C) Both 1 and 2
record shows little signs of improvement. Why is (D) Neither 1 and 2
this so? Before examining this and looking at some
possible solutions, here’s an anecdote. An Indian 7. A three-member panel was set up in April 2014 on
family travelling in Switzerland asked the driver of
road safety. Who headed the panel?
their minibus to stop by the road so that their young
(A) TV Raveendran
daughter could relieve herself. Although they were
on a local highway in a suburban area with negligible (B) KS Radhakrishnan
traffic, the driver refused to stop, saying: “I can’t (C) K Kasturiranagan
stop here. A traffic judge would ask, what’s more (D) Ranjana Desai
important, someone wetting the seat or the safety of
those on the road? We need to wait till we reach a 8. Which of the following measures plays a crucial role
public convenience.” The family continued pestering in significantly reducing road accidents in India?
him, “It will not take more than a minute…” In India,
(A) Raising awareness through public campaigns
we seem to be hundreds of years away from the
and advertisements
mindset of that minibus driver, as seen in the rising
number of road crashes. Looking at the gross (B) Increasing penalties for traffic violations
violations of the traffic norms and values in (C) Implementing automated speed detection
important cities and upcoming towns in the country systems
one can infer that road safety is nobody’s baby: It is (D) Developing a robust emergency response
an orphan, desperately waiting for adoption. Deadly system
violations of lane driving, speed limits, and traffic
signals, instances of at-will parking on the fast-
9. “Hit and Run motor accident” is defined under which
developing modern, smooth highways — all these go
mostly unchecked and unquestioned. The causes of section of Motor Vehicles Act, 1988?
road crashes, such as the ones above, are well (A) Section 160
known. Human error on the roads is admittedly the (B) Section 161(1)(b)
single largest factor responsible. How are we so (C) Section 164
helpless against this man-made menace, while a (D) Section 159
pandemic caused by a virus is being speedily tamed?
The enforcement of traffic norms is the key to road
safety. This is not to simplify the complex nature of
10. Which of the following statements is/are true about
the problem at hand. All ongoing programmes
towards enhancing safe road conditions and vehicles NCRB?
have to go on. However, the priority goal and the 1. It was set up in 1995 to function as a repository
global mandate is to significantly reduce the rising of information on crime and criminals.
number of road crashes. The central and state 2. Ramphal Pawar is the current Director of the
governments run complex road safety programmes National Crime Record Bureau.
with their scarce resources, with little success. The 3. It works under the Ministry of Law and Justice.
World Bank has chipped in with a $250 million loan
(A) 1 and 2
to India to tackle the high rate of road crashes
(B) 1 and 3
through road-safety institutional reforms and results-
based interventions. (C) 2 and 3
(D) None of the above

[17]
PASSAGE III Parliamentarians for certain legislative amendments
For the second time in a decade, the U.N.-recognised to improve compliance with the Paris Principles.
Global Alliance of National Human Rights
Institutions (GANHRI) deferred the accreditation of 11. What are the Paris Principles?
National Human Rights Commission, India (NHRC- (A) International benchmarks against which NHRIs
India) citing objections like political interference in can be accredited
appointments, involving the police in probes into
(B) A set of guidelines for conducting
human rights violations, and poor cooperation with
investigations into human rights violations
civil society. As reported, the GANHRI’s letter to
the NHRC also cited lack of diversity in staff and (C) A UN resolution on the protection of human
leadership, and insufficient action to protect rights defenders
marginalised groups, as reasons for the deferment of (D) A list of criteria for selecting members of
the accreditation. This came two months after seven national human rights commissions
human rights watchers/institutions wrote to
GANHRI objecting to NHRC India’s ‘A’ rank. They 12. Why did the GANHRI defer the accreditation of the
also raised concerns regarding the commission’s
NHRC?
lack of independence, pluralism, diversity and
(A) Lack of diversity in staff and leadership
accountability that are contrary to the U.N.’s
principles on the status of national institutions . “The (B) Insufficient action to protect marginalized
NHRC has repeatedly failed to deliver its mandate, groups
in particular to protect the rights of people from (C) Involving the police in probes into human rights
marginalized communities, religious minorities, and violations
human rights defenders,” the letter said. The United (D) All of the above
Nations’ Paris Principles, adopted by the U.N.
General Assembly, provide the international
13. What are the six main criteria that NHRIs are
benchmarks against which National Human Rights
required to meet under the Paris Principles?
Institutions (NHRI) can be accredited.
The Paris Principles set out six main criteria that (A) Mandate and competence
NHRIs are required to meet. The letter, written by (B) Autonomy from government
Amnesty International, CSW, Front Line Defenders, (C) Independence guaranteed by a statute or
Human Rights Watch International Commission of Constitution
Jurists, International Federation for Human Rights (D) All of the above
(FIDH), and the World Organisation Against Torture
(OMCT) to GANHRI, maintained that since 2017, 14. What is ‘A’ status?
instead of following through on the assurances that (A) The highest rating given by the GANHRI to
the NHRC has given to the GANHRI, the NHRIs that follow the Paris Principles
functioning of the commission has further regressed, (B) A category of human rights violations
undermining its independence and adherence to the recognized by the UN
Paris Principles. Responding to The Hindu, the
(C) A level of accreditation for international NGOs
NHRC-India said that the GANHRI, through the
working on human rights issues
Sub-Committee on Accreditation (SCA) is
(D) A classification for countries that have achieved
responsible for reviewing and accrediting NHRIs in
significant progress in protecting human rights
compliance with the Paris Principles every five
years. As part of this process, the review of the
15. What are the Paris Principles?
NHRC-India was due in March 2023 for its
reaccreditation, which has been deferred for a year, (A) International benchmarks against which NHRIs
meaning thereby no final decision has been taken as can be accredited
yet. The NHRC said that the deferment is an action (B) A set of guidelines for conducting
of the SCA and not a recommendation on the investigations into human rights violations
accreditation status of the NHRC-India. It added that (C) A UN resolution on the protection of human
throughout the deferred period, India’s top human rights defenders
right’s body added that the process of reaccreditation (D) A list of criteria for selecting members of
has not concluded and the SCA has recommended national human rights commissions
advocating with the government and
[18]
PASSAGE IV 18. What percentage of total electricity generated in
The need for additional capacity of 16,900 MW has India comes from coal?
been cited in the report of the Central Electricity (A) 40% (B) 50%
Authority (CEA) called Optimal Generation (C) 60% (D) 70%
Capacity Mix, which was released recently. In fact,
there are two versions of this report. The first was 19. What is the main disadvantage of continuing to
published in January 2020 and the second in April operate generating units that are more than 25 years
2023. So what’s the catch? How are we to meet our old?
demand in 2029-30 if no new coal-based plants are (A) They are more expensive than renewable
allowed? This writer’s surmise is that the energy options
government probably feels that the additional (B) They adversely affect the station heat rate of
capacity of about 16,000 MW of coal-based capacity plants
may ultimately not be required, primarily for two (C) They require large quantities of water for
reasons. The first reason is the demand for power in cooling and other processes
2029-30 in the second report is based on the 20th (D) They do not have existing transmission links or
Electric Power Survey (EPS) whereas the first report coal linkages
looked at the estimates of the 19th EPS. The demand
for power in 2029-30 in the 20th EPS is somewhat 20. What is the main advantage of diversifying India's
more conservative. This is understandable since the energy sources?
estimates get closer to reality once we approach the (A) Reducing pollution from coal combustion
terminal date. (B) Increasing the cost of electricity generation
In the second version of the CEA report, the required (C) Reducing India's dependence on coal imports
capacity for coal-based stations in 2029-30 has come (D) Enhancing grid reliability and resilience
down from 267 GW to 251 GW. This decrease is on
account of the reasons mentioned above and also PASSAGE V
because of a major change in policy relating to the NTPC signed a supplementary joint venture
retirement of units after they complete 25 years. This agreement with the Nuclear Power Corporation of
version mentions that about 2,121 MW of coal-based India on Monday to develop nuclear electricity
capacity would be retired by 2030 whereas the
generation projects.
earlier version of this report stated that about 25,000
The joint venture will initially develop two
MW of coal-based capacity would be retired by
2030. The government probably feels it would be a pressurized heavy-water reactor projects - 1,400
good idea to carry on with old plants even after they MW Chutka in Madhya Pradesh and 2,800 MW
have completed 25 years of operation. Carrying on Mahi Banswara in Rajasthan. The largest power
with generating units that are more than 25 years old generator in India aspires to become a 130-GW
is not a bad idea since the station heat rate of well- company by 2032 of which 2,000 MW will be from
maintained plants does not get adversely affected nuclear sources. However, the entire capacity of the
with age. The advantages of carrying on with old
two projects may come online by 2035, the company
plants are that the transmission links are already
had earlier said. "We are confident that out of the
there and that the coal linkages are maintained.
However, such plants should sell their power in Chutka and Mahi Banswara, we will be definitely
exchange instead of signing fresh PPAs. completing 2,000 MW," it had said. Currently,
NTPC has a total installed power generation capacity
16. What is the main reason for India's decision to stop of 71.644 GW, of which 3.01 GW is renewable
building new coal-fired power plants? energy.
(A) To improve air quality and public health
(B) To reduce India's dependence on coal imports
21. Which of the following is a joint venture project with
(C) To align with global trend of phasing out coal
Russia?
(D) To increase the cost of electricity generation
(A) Kudankulam Nuclear Power Plant
17. What is India's proposed renewable energy capacity (B) Farakka Super Thermal Power Plant
target by 2030? (C) Bongaigaon Thermal Power Station
(A) 500 MW (B) 500 GW (D) Meja Thermal Power Project
(C) 500 KW (D) 500 TW
[19]
22. Which government body is responsible for the safety dissent, is currently on hold due to a continuing
regulation of NPCIL's nuclear power plants? interim order of the top court passed on May 11,
(A) NPCIL Safety Committee 2022.
(B) Indian Nuclear Regulatory Commission On Monday, the AG’s submissions came as the court
(C) Atomic Energy Regulatory Board (AERB) mulled whether it should consider a larger bench for
determination of the validity of the sedition law since
(D) Nuclear Power Plant Safety Authority
a five-judge bench had in 1962 upheld the legality of
Section 124A.
23. National thermal power corporation comes under Venkataramani, on his part, suggested that the court
which ministry? should defer its decision on the strength of an
(A) Ministry of Petroleum and Natural Gas appropriate bench to hear the bundle of petitions that
(B) Department of Atomic Energy have challenged the validity of the sedition law in the
(C) Ministry of Coal wake of the ongoing exercise by the government.
(D) Ministry of Power (WITH EDITS and revisions the telegraph)

24. Which of the following is not a Central Public Sector 26. Which of the following is the section is related to
Undertaking (PSU) in India? sedition?
(A) NTPC Limited (A) 124 A (B) 127 A
(B) Air India (C) 129 A (D) 144 A
(C) Bharat Sanchar Nigam Limited
(D) Hindustan Copper Ltd. 27. The sedition law in the IPC has originally been
drafted by whom?
25. Who is The Union Minister of State for Atomic (A) Lord Canning
Energy and Space in India?
(B) Lord Curzon
(A) Ajay Kumar Mishra
(C) Thomas Macaulay
(B) Jintendra Singh
(C) V. Muraleedharan (D) Vincent Chase Sedict
(D) Arjun Ram Meghwal
28. What was the first case to be tried under the law of
PASSAGE VI sedition in India?
The Supreme Court in a historic judgement (A) Queen-Empress v Bal Gangadhar Tilak
effectively put on hold the colonial-era penal (B) Queen-Empress v Jogendra Chunder Bose
provision, until the Union government reviews it.
The legislative process of reviewing the 153-year- (C) Queen-Empress v Mahatma Gandhi
old colonial law on sedition in India is in the “final (D) None of the above
stages”, the Union government on Monday informed
the Supreme Court, adding “the government is keen 29. What was the first country to implement sedition
on pushing reforms” and that something may be in law?
works as early as the coming monsoon session of
Parliament. (A) Britain
Appearing before a bench headed by Chief Justice of (B) United states of america
India (CJI) Dhananjaya Y Chandrachud, attorney (C) India
general R Venkataramani submitted that the (D) France
government has set up a committee to review Section
124A of the Indian Penal Code and that the
deliberation with the stakeholders was on. 30. In which case the supreme court held that “criticism
“After the order of this court on May 11, 2022, the of the government exciting disaffection or bad
process of re-examination has been set in motion. feelings towards it, is not to be regarded as a
The consultation among all the stakeholders is in the justifying ground for restricting the freedom of
final stages. After that, it will go to Parliament. But expression and of the press, unless it is such as to
before it goes to Parliament, it will be shown to me.
The government is keen on pushing reforms,” undermine the security of or tend to overthrow the
Venkataramani told the bench, which also comprised state.”?
justice JB Pardiwala. (A) Tara Singh Gopi Chand v. The State (1951)
The operation of Section 124A in the IPC — a non- (B) Kedarnath Singh v State of Bihar
bailable offence punishable with jail term ranging (C) Romesh Thapar v State of Madras
from three years to life, and one that activists and
jurists have alleged is often misused to muzzle (D) Ram Nandan v. State of Uttar Pradesh

[20]
Legal Reasoning
PASSAGE I One of the spouses involved in the intercourse with
In Ancient times, the concept of divorce was not another person, married or unmarried, of the
known to anyone. They considered marriage as a opposite sex.
sacred concept. According to Manu, the husband and Intercourse should be voluntary and consensual.
wife cannot be separated from each other, their At the time of the act, the marriage was subsisting.
martial tie cannot be broken. Later the concept of There must be sufficient circumstantial evidence to
divorce came in the picture and established as a prove the liability of another spouse.
custom to put the marriage to an end. Cruelty
According to the Arthashastra, marriage can end if The concept of cruelty includes mental as well as
dissolved by mutual consent and should be physical cruelty. The physical cruelty means when
unapproved marriage. But Manu does not believe in one spouse beats or causes any bodily injury to the
the concept of the dissolution. According to Manu other spouse. But the concept of mental cruelty was
the only way to end the marriage is the death of one added as the spouse can also be mentally tortured by
of the spouses. the other spouse. Mental Cruelty is lack of kindness
The provision related to the concept of divorce was which adversely affects the health of the person.
introduced by the Hindu Marriage Act, 1955. The Well it is easy to determine the nature of physical
Hindu Marriage Act defines divorce as a dissolution cruelty but difficult to say about mental cruelty
of the marriage. For the interest of the society, the Desertion
marriage or the marital relationship needs to be Desertion means the permanent abandonment of one
surrounded by every safeguard for the cause spouse by the other spouse without any reasonable
specified by law. Divorce is permitted only for a justification and without his consent. In General, the
grave reason otherwise given other alternative. rejection of the obligations of marriage by one party.
The Hindu Marriage Act is based on the fault theory
in which any one of the aggrieved spouses (Section 1. Dhinchan and Paro met at a Moot court competition
13(1)) can approach the court of law and seek the at NALSAR and fell in love and decided to get
remedy of divorce. Section 13(2) provides the married against the wishes of their family. after 5
years their marriage turned sour, paro suspects the
grounds on which only the wife can approach the
Dhinchan has an extra maritial affair with his male
court of law and seek the remedy of divorce Grounds
coworker suresh. one day she finds them in
for divorce- objectional position . she decides to file for divorce
Adultery on the ground of adultery. Is she likely to succeed?
The concept of Adultery may not be considered as (A) Yes, since Dinchan is cheating on her with
an offence in many countries. But as per the Hindu suresh while marriage was subsisting and
Marriage Act, in the matrimonial offence, the adultery is considered as one of the most
adultery is considered as one of the most important important ground for seeking divorce
(B) Yes, since adultery is considered as one of the
ground for seeking divorce. Adultery means the
most important ground for seeking divorce.
consensual and voluntary intercourse between a
Adultery means the consensual and voluntary
married person with another person, married or intercourse between a married person with
unmarried, of the opposite sex. Even the intercourse another person
between the husband and his second wife i.e. if their (C) No, since there is no proof that the intercourse
marriage is considered under bigamy, the person is for consensual and Adultery means the
liable for the Adultery. consensual and voluntary intercourse between a
The concept of Adultery was inserted under the married person with another person
(D) No, since suresh is a Man and the intercoure
Hindu Marriage Act by the Marriage Laws
should be between person of opposite sex
Amendment Act, 1976.
Essentials of Adultery

[21]
2. Paul and Meagan have been married for four and a (C) Suresh has Extra marital affairs with Priya
half year. one day a plumber comes to their house to (D) both A and B
fix a leaking pipe While working, the plumber and
Meagan strike up a friendly conversation and 5. Pranil is owner of a multibillion dollar company in
mutually decide to engage in sexual activities. paul India. he is a workoholic person he works day and
discovers about this and files for divorce of the night his only goal is to become the richest person in
grounds of adutery and he has a substantial proof. Is India but he also enjoys beating his assistant in free
he likely to succeed? time. one day he beats her with an iron rod and
(A) No, since there is no proof that the intercourse sexually abuses her. his wife founds out about this
for consensual and Adultery means the and decides to file for divorce on grounds of cruelty
consensual and voluntary intercourse between a as he might do same things with her. Is she likely to
married person with another person. plumber succeed?
might have forced her (A) No, since abusive behavior towards the assistant
(B) Yes, since adultery is considered as one of the does not directly prove that he will do same with
most important ground for seeking divorce. his wife
Adultery means the consensual and voluntary (B) Yes, since physical and sexual abuse towards
intercourse between a married person with the assistant demonstrates a pattern of cruelty,
another person he might do same kind of abuse with her
(C) Yes, since at that time marriage was subsisting, (C) It depends on the available evidence and the
Maegan had intercourse with the opposite sex discretion of the court to determine if the wife's
and adultery is considered as one of the most concerns about potential abuse are justified and
important grounds for seeking divorce if they warrant granting a divorce on grounds of
(D) No, since Paul is a Man and a Man can't file for cruelty
divore under Hindu marriage act,1955 (D) No, since abusive behavior towards the assistant
does not directly prove that he will do same with
3. Pramanita and lafik married each other 10 years ago. his wife and cruelty means when one spouse
after 7 years lafik realized that he can't live with beats or causes any bodily injury to the other
pramanita because he believed that she is unlucky for spouse which is not the case here
him and decided to live separately without taking her
permission. can pramanita get divorce on grounds of PASSAGE II
desertion? Article 19(1)(a) of the Indian Constitution grants
(A) Yes, since lafik left her without any reasonable every citizen the right to free speech and expression.
justification and without her consen This right encompasses both the dissemination and
(B) No, since lafik left her with reasonable receipt of information, as being adequately informed
justification and without her consent is essential for forming informed opinions.
(C) No, since lafik left her with reasonable Therefore, the right to seek and acquire information
justification and with her consent is an integral aspect of the fundamental right
(D) Yes, since lafik just went for a vacation and had protected by Article 19(1)(a). The Right to
not abandoned her Information (RTI) Act of 2005 aims to achieve
several objectives:
Establishing a practical framework: The RTI Act
4. Reshma and Suresh have been married for 15 years.
provides a framework that enables citizens to access
Recently, Suresh lost his job due to pora-19
information held by public authorities.
pandemic and stopped giving her luxury gifts. She
Promoting transparency and accountability: The Act
feels neglected by Suresh so she starts having
aims to enhance transparency and accountability in
intercourse with Rohit who gives her many luxury the functioning of government bodies and related
gifts. Meanwhile Suresh also starts having Extra entities.
Marital affairs with Priya, Sister of Rohit. Hamina, Establishing Information Commissions: The Act
wife of Rohit files for divorce on grounds of provides for the creation of Information
adultery. Which among the following arguments can Commissions at both the state and national levels.
make her case strong? These commissions are responsible for executing the
(A) Intercourse between Rohit and Reshma are functions and exercising the powers outlined in the
consensual Act.
(B) At the time of the act, the marriage was Developing an informed citizenry: By granting
subsisting access to information, the Act seeks to foster an
[22]
informed citizenry capable of participating in (C) No, because the citizen has the right to access
democratic processes. information about the utilization of public
Combating corruption: The Act aims to curb funds.
corruption by facilitating access to information and (D) No, because the PIO is obligated to disclose
promoting accountability within public authorities. information unless it falls within the specified
Balancing conflicting interests: The Act delineates exemptions.
exemptions to the disclosure of information in cases
where such disclosure may conflict with other public 7. A public authority receives an application seeking
interests. It seeks to harmonize these conflicting information about an ongoing investigation. The
interests while upholding the paramountcy of
authority denies the request, claiming that it could
democratic ideals.
impede the investigation process. Is the denial
Section 4(1) of the Act imposes specific obligations
justified under the RTI Act?
on public authorities, including the maintenance of
(A) Yes, because the disclosure of information
records. Public authorities are required to organize
about ongoing investigations can hinder the
and index their records and, where feasible,
computerize them for ease of access. They must also investigative process.
publish certain particulars within 120 days of the (B) Yes, because protecting the integrity of ongoing
Act's enactment. investigations is of higher importance than the
Section 4(2) mandates that public authorities right to information.
proactively provide information to the public (C) No, because citizens have the right to access
through various means of communication at regular information unless it falls within the specified
intervals. exemptions.
Sections 4(3) and 4(4) focus on the dissemination of (D) No, because the RTI Act does not provide
information. Section 4(3) emphasizes the wide exemptions for ongoing investigations.
dissemination of information in an easily accessible
manner for the public. Section 4(4) outlines factors 8. A public authority failed to publish certain
to consider during information dissemination, such particulars within the prescribed timeframe of 120
as cost-effectiveness, using the local language, and days. Can a citizen file a complaint against the
selecting the most effective communication method authority under the RTI Act?
for a particular area. (A) Yes, because the public authority has a legal
Section 5 of the Act pertains to the designation of obligation to publish the particulars within the
Public Information Officers (PIOs). Public specified timeframe.
authorities must appoint Central and State PIOs
(B) Yes, because the citizen has the right to hold the
within 100 days of the Act's enactment. These
public authority accountable for non-
officers have the responsibility to provide
compliance with the RTI Act.
information requested under the Act. Additionally,
(C) No, because the RTI Act does not provide for
Section 5(2) calls for the designation of Assistant
complaints against public authorities.
PIOs at lower administrative levels to receive
applications and appeals and forward them to the (D) No, because the prescribed timeframe of 120
appropriate authorities. days is not mandatory but only a suggested
In summary, the RTI Act of 2005 was enacted to guideline.
ensure access to information, promote transparency
and accountability, combat corruption, and foster an 9. A citizen submits an application seeking information
informed citizenry. from a public authority but does not receive a
response within the specified timeframe. Can the
6. A citizen filed an application seeking information citizen file an appeal under the RTI Act?
about the utilization of public funds by a government (A) Yes, because the citizen has the right to appeal
department. The Public Information Officer (PIO) if the public authority fails to respond within the
denied the request, stating that it would adversely specified timeframe.
affect the economic interests of the country. Was the (B) Yes, because the citizen has the right to access
PIO's decision justified under the RTI Act? information and seek remedies for non-
(A) Yes, because disclosure of such information compliance with the Act.
could harm the economic interests of the (C) No, because the RTI Act does not provide for
country. the right to appeal in case of non-response from
(B) Yes, because economic interests take public authorities.
precedence over the right to information.
[23]
(D) No, because the specified timeframe for  With consent obtained by putting her or any other
response is not legally binding and only serves person in whom she is interested under fear of
as a guideline. death or of hurt;
 With consent given under a misconception of fact
10. A public authority receives a request for information that the man was her husband.
but refuses to provide it, citing the lack of resources  Consent is given by the reason of unsoundness of
to disseminate the information effectively. Is the mind, or under the intoxication of any stupefying
public authority's refusal justified under the RTI or unwholesome substance;
Act?  Women under the age of eighteen with or without
(A) Yes, because the public authority is not obliged consent;
to disseminate information if it lacks the  When a woman is unable to communicate
necessary resources. consent.
(B) es, because the cost-effectiveness and Gang Rape
effectiveness of information dissemination Section 376D of the IPC defines gang rape as the
methods are important considerations. rape committed by one person or more than one
(C) No, because the RTI Act mandates public constituting a group or acting in furtherance with the
authorities to disseminate information in an common intention and each of those persons shall be
easily accessible manner. liable for the imprisonment for not less than twenty
(D) No, because the public authority must allocate years which can be extended to imprisonment for life
resources to ensure effective information and they would also be liable for fine and the fine
dissemination. will be reasonable to the medical expenses of the
victim and the rehabilitation of the victim. As stated
PASSAGE III above, earlier the punishment was for imprisonment
Sexual offences as we all know are basically the for not less than ten years was there but was later
offences related to physically harm the private or the changed after The Criminal Law Amendment, 2013
sexual parts of a woman. These offences include to imprisonment not less than twenty years
offences like rape, child abuse, gang rape, sexual
harassment etc. 11. Uttam, first year student of NALSAR is attracted to
the definition of the term “Rape” under Section 375 Raji, professor of law of torts. He proposed her one
of IPC is defined as “A man is said to commit a rape day but she refused so he decided to take revenge and
if he penetrates his anus into any extent into the decide to rape her. At 12:10 am he enters her room
vagina, mouth, urethra or anus of a woman or inserts and puts himself on her and applies his mouth on her
any object, to any extent, or any part of a body other vagina but he is stopped by vice chancellor just
before he was about to penetrate her vagina. Raji
than the anus into the vagina, urethra and anus of a files a case against him Under section 375 You’re the
woman or applies his mouth into the vagina, urethra district judge decide the case
or anus of a woman or makes her to do so with him (A) Uttam is not liable under section 375 because he
or any other person”. loves her and had no intention to hurt her
definition is further subjected to the several (B) Uttam is liable under section 375 as he applied
circumstances like if the act is done against her will his mouth on her vagina which comes under
rape
or without her consent or if the consent is taken by
(C) Uttam is not liable under section 375 as he was
coercion or if the consent is obtained by fraud or if not able to penetrate her vagina and penetration
the consent is obtained when the girl was in is essential for rape
intoxication or was unsound at that time or obtaining (D) Uttam is liable under section 376 as he applied
the consent of a girl which is not able to his mouth on her vagina which comes under
communicate and the most important when the act is rape
done with or without the consent of a girl below 18
12. Raja, a Professor at National law university
years of age
dholakpur is famous for his habit of excessive
Essential ingredients drinking one day he drinks too much and goes to
As stated above, there are basically seven essential Rohan’s room. Rohan is a biological male but due to
ingredients to constitute a rape- a lack of testosterone he behaves like a woman. He
 Against her will; was so intoxicated that he thought Raman is his wife
 Without her consent; and puts his finger inside his anus. Raman files a case
of rape under section 375 against Raja. decide the
case

[24]
(A) Raja is not liable under section 375 as he was (C) Susmita should file case Under section 376 also
not able to penetrate his vagina and penetration her consent was not valid as she gave her
is essential for rape consent under threat and maximum punishment
(B) Raja is liable under section 375 because he is life Imprisonment
penetrated his anus which comes under rape and (D) Susmita should file case Under section 376 also
Rohan behaves like a Woman her consent was not valid as she gave her
(C) Raja is not liable as he was intoxicated and consent under threat and maximum punishment
thought Rohan is his wife is 20 years
(D) Raja is not liable because Rohan is not a woman
and rape can only be committed against a PASSAGE IV
women The Right to Freedom of Religion is guaranteed to
all Indians by the Constitution under Articles 25 to
13. Vladimir, a Russian student who is 18 years old
came to NLIU with his girlfriend Natalia who was 28. This topic, which is intertwined with the concept
17 years and 11 months old on 2nd December 2022. of secularism is very important.
They both orally decided to have sex on 3rd January Right to Freedom of Religion
2024. Vice chancellor files a case of rape against The Constitution of India guarantees the right to
Vladimir. You are CJM jony pins decide the case freedom of religion to not only individuals but also
(A) Vladimir is liable Under section 375 because He religious groups in India. This is enshrined in
had sex with Natalia who is 17 years old
Articles 25 to 30.
(B) Vladimir is liable Under section 375 because He
had sex with Natalia without her written consent Article 25 (Freedom of conscience and free
(C) Vladimir is not liable as she is above the age of profession, practice, and propagation of religion)
18 Article 25 guarantees the freedom of conscience, the
(D) Vladimir is not liable as she is above the age of freedom to profess, practice, and propagate religion
18 and her consent is valid to all citizens.
 The above-mentioned freedoms are subject to
14. Roman, Pinku, Dinku, third year student of PNLSIU
Hate Susmita and one day decide to rape her with public order, health, and morality.
common intention. They enter her room at 5 pm and  This article also gives a provision that the State
show her their gun and ask her if they can have sex can make laws:
with her, she agrees. Susmita files a case against  That regulates and restricts any financial,
them under section 376. She comes to you to ask for economic, political, or other secular activity
legal advice and asks what is the maximum
associated with any religious practice.
punishment they can get. What is your advice to her?
(A) Susmita should file case under section 375 and  That provides for the social welfare and reform
maximum punishment is 10 years or opening up of Hindu religious institutions of a
(B) Susmita should file case Under section 376 also public character to all sections and classes of
her consent was not vaild as she gave her Hindus. Under this provision, Hindus are
consent under threat and maximum punishment construed as including the people professing the
is 20 years
Sikh, Jain, or Buddhist religions, and Hindu
(C) Susmita should file case Under section 376 also
her consent was not vaild as she gave her institutions shall also be construed accordingly.
consent under threat and maximum punishment  People of the Sikh faith wearing & carrying the
is life Imprisonment kirpan shall be considered as included in the
(D) Susmita should not file any case against them as profession of the Sikh religion.
it was consensual case and there was no Article 26 (Freedom to manage religious affairs)
apprehension of threat This Article provides that every religious
denomination has the following rights, subject to
15. Roman, Pinku, Dinku, third year student of PNLSIU
Hate Susmita and one day decide to have sex with morality, health, and public order.
her. They enter her room at 5 pm and give her The right to form and maintain institutions for
precious stones and ask her if they can have sex with religious and charitable intents.
her, she agrees. Susmita files a case against them The right to manage its own affairs in the matter of
under section 376. She comes to you to ask for legal religion.
advice and asks what is the maximum punishment
The right to acquire the immovable and movable
they can get. What is your advice to her?
(A) Susmita should file case under section 375 and property.
maximum punishmentnt they can get is 10 years The right to administer such property according to
(B) Susmita should not file any case against them as the law.
it was consensual and there was no
apprehension of threat
[25]
Article 27 (Freedom as to payment of taxes for law prohibiting this citing health concerns. Some 5-
promotion of any particular religion) year law students of Palsar University challenge this
According to Article 27 of the Constitution, there law. You’re the counsel for the state of Madhya
can be no taxes, the proceeds of which are directly Pradesh. Which among the following is the reason
used for the promotion and/or maintenance of any you can give in court defending the decision of state
particular religion/religious denomination. of Madhya Pradesh
Article 28 (Freedom as to attendance at religious (A) This law is Valid as sacrificing children is an
instruction or religious worship in certain immoral act and article 25 is subject to
educational institutions) Abhijeet's morality
This article permits educational institutions that are (B) This law is Valid as sacrificing children is an
maintained by religious groups to disseminate immoral act and article 25 is subject to Public
religious instruction. Health
 This provides that no religious instruction shall (C) This law is Valid as sacrificing children is an
be provided in State-run educational institutions. immoral act and article 25 is subject to
 Educational institutions administered by the State community health
but that were established under any endowment (D) All the above
or trust which requires that religious instruction
shall be imparted in such institutions are exempt 18. PCN community is a minority community in cest
from the above clause (that no religious pengal state. Chief minister of the state hates them
instruction shall be provided). and stops them from dissipating religious knowledge
 Any person who attends any educational in educational institutions managed by the
institution recognized by the State or receiving community. Member of the PCN community want to
State aid shall not be required to participate in any file a case for violation of their constitutional rights.
religious instruction that may be imparted in such You are their lawyer tell which right is violated here?
institution, or also attend any religious worship in (A) religious rights under article 25 are violated
such institutions unless he/she has given consent here as it permits educational institutions that
for the same. In the case of minors, the guardians are maintained by religious groups to
should have given consent for the same disseminate religious instruction
(B) religious rights under article 26 are violated
16. INIM is a religious organization in Madhya Pradesh here as it permits educational institutions that
state in a country called Ifia which practices a are maintained by religious groups to
practice of throwing Childrens in open fire as a part disseminate religious instruction
of their religious ceremony. Chief minister of (C) religious rights under article 27 are violated
Madhya Pradesh Abhijeet Singh Passes a law here as it permits educational institutions that
prohibiting this practice citing children rights. Many are maintained by religious groups to
communist organizations protest and challenge these disseminate religious instruction
laws in supreme court. You’re the chief justice of (D) religious rights under article 28 are violated
Supreme court decide the case here as it permits educational institutions that
(A) The law is invalid as it unjustifiably infringes are maintained by religious groups to
upon the right to freedom of religion protected disseminate religious instruction
under article 25 state should stay away from
religious matters 19. Pota is a minority religious community in Punjab
(B) This law is Valid as sacrificing children is an province of British Raj which practices drinking
immoral act and article 25 is subject to morality blood of humans. Governer of Punjab Lord Promod
(C) Abhijeet Singh is a fascist because he prohibits this citing Human rights. Members of the
unjustifiably infringes upon the right to freedom religious community challenge the law, arguing that
of religion of INIM organization it infringes upon their right to freedom of religion
(D) both A and C under Article 25. decide the case
(A) This law is valid as religious freedoms under
17. INIM is a religious organization in Madhya Pradesh article 25 are subject to some restrictions
state in a country called Ifia which they consume (B) This law is valid as religious freedoms under
Drugs as a part of their religious ceremony. Chief article 25 are subject to morality and this
minister of Madhya Pradesh Abhijeet Singh Passes a practice is Immoral

[26]
(C) This law is Invalid as religious freedoms under for performance of valid marriage. If anyone of the
article 25 are not subject to any restrictions bride or groom is below the legal age, the marriage
(D) none of the above solemnized is void
Prohibited Degrees of Relationship:
20. Pontie community of Madhya Pradesh wants to Sec. 5(iv) of Hindu Marriage Act,1955 says that no
purchase an immovable property but they were marriage can be solemnized falling within prohibited
stopped by a local MLA as he uses this property for degree of relationship. Those marriage which are
urinating. Pontie community files a case for violation solemnized under this they all are void under section
of rights. You are CJM ria Khalifa decide the case 11 of Hindu Marriage Act, 1955
(A) There is violation of Article 26 (Freedom to Prohibition of Sapinda Relationship:
manage religious affairs) which includes Sec. 5(v) of Hindu Marriage Act, 1955 says that no
purchase of immovable property one can marry with their Sapinda relationship person
(B) There is violation of Article 25 (Freedom to it is an offence under the section 11 of the act. In the
manage religious affairs) which includes sapinda relationship third generation in the line of
purchase of immovable property ascent through the mother and fifth in the line of
(C) There is violation of Article 28 (Freedom to ascent through the father
manage religious affairs) which includes
purchase of immovable property 21. Which offense is punishable under the Indian Penal
(D) Article 26 (Freedom to manage religious Code in case of more than 2 marriages without
affairs) which includes purchase of immovable divorce?
property is subject to reasonable restriction and (A) Polyandry
this restriction is reasonable (B) sologamy
(C) Tologamy
PASSAGE V (D) Bigamy
Marriage is a social practice through which two
people and their families unite which gives rise to 22. Jungkook, 18-year-old first year student of NLIU
conjugal rights. The word conjugal means rights wants to marry love of his life Rashi who is also 18
which arise between husband and wife after years old. Is their marriage valid under Hindu
marriage. Marriage is one of the oldest sacraments in marriage Act?
the society. (A) No, they have not reached the legal age of
Marriage is a universal social institution. It brings marriage as legal age for marriage for boys is 21
men and women into family life. In this institution (B) Yes, they have reached the legal age of marriage
man and woman are socially permitted to have as legal age for marriage as legal age for
children. Thereby society grants. Them the rights to marriage is 18 years
sexual relation. Marriage is a socially approved way (C) No, they have not reached the legal age of
of establishing a family of procreation marriage as legal age for marriage for girls is 21
Essential element of marriage. (D) Yes, they have reached the legal age of marriage
Monogamy as they love each other
Monogamy is the marriage between one woman and
one man at a time. Monogamy is the most sacred 23. Which among the following marriage is prohibited
type of marriage in Hindu. Sec. 5(i) of Hindu under Sec. 5(v) of Hindu Marriage Act, 1955?
Marriage Act says that only monogamy is accepted (A) a marriage with third generation in the line of
in Hindu religion and if spouse of any one is not dead ascent through the mother
at the time of marriage, then that would be (B) a marriage with fifth in the line of ascent
considered as the offence of bigamy under Hindu through the father
Marriage Act, 1955. (C) a marriage with third in the line of ascent
The offence of bigamy is also punishable under through the father
section 494 of Indian Penal Code, 1860 (D) both A and B
Age to the parties:
According to Hindu Marriage Act, 1955 the legal 24. Depesh is in love with Suman. They want to marry
age for male is 21 years and for the female is 18 each other but don’t know what is the purpose. They
years. If the given age is not performed by any of the come to you to for advice. What is the purpose of
party, then it is an offence under Hindu Marriage marriage according to you?
Act, 1955. Age is one of the most important essential
[27]
(A) Marriage is a social practice through which two Act requires “suo moto disclosure of information”
people and their families unite which gives rise by every public authority. However, what this
to conjugal rights section deals with has some-what not become
(B) society grants Them the rights to sexual relation satisfactory.
(C) Marriage is a socially approved way of Section 6 – It simply prescribes an easy procedure
establishing a family of procreation for securing information through the Act.
(D) all the above Section 7 – This section has prescribed a time for the
PIO’s to give out the information requested by the
25. Nobita married Shizuka 10 years ago their marriage public. It gives a time frame of 30 days in most cases.
Section 8 – This section and all subsections that are
turned sour and Nobita filed for divorce. While
contained in it describe how only minimum
divorce proceedings were going on he married Tia
information may be exempted from disclosure to the
Khalifa. Shizuka claims that it is bigamy. You are
public. It described what all may be exempted in a
her lawyer tell the reason to the court
few cases, etc.
(A) Sec. 5(i) of Hindu Marriage Act says that only
Section 19 – Completely described the two-tier
monogamy is accepted in Hindu religion and if mechanism that exists for appeal.
spouse of any one is not dead at the time of Section 20 – Provides penalty fees, etc. in case there
marriage, then that would be considered as the is a delay in giving out information or the
offence of bigamy information is distorted or wrong in any situation.
(B) Sec. 4(i) of Hindu Marriage Act says that only Section 23 – This states that the lower courts are
Bigamy is accepted barred from entertaining any suits or applications in
(C) Sec. 5(i) of Hindu Marriage Act says that you this regard. However, the writ jurisdiction of the
cannot divorce your wife Supreme Court and High Courts are not affected in
(D) none of the above any way. They remain the same, under article 32 and
225 of the Constitution.
PASSAGE VI
The Right to Information (RTI) Act, 2005 came into 26. A private company receives funding from the central
force on the midnight between 12th and 13th government for a specific project. Is the private
October. he RTI Act is also referred to as a Central company considered a public authority under the
Legislation “to provide for setting out the particular RTI Act, 2005?
regime of right to information for citizens”, it (A) Yes, because the RTI Act includes civil
enables Indians to exercise their Fundamental Right societies indirectly funded by public funds as
of Speech and Expression. Section 1(2) – It is clearly public authorities.
stated here that the RTI Act extends to the whole (B) Yes, because the private company is carrying
nation, except for the State of Jammu and Kashmir, out a project under the monitoring of the central
as it has its state-level governance. government.
Section 2 (f) – This section exhaustively defines the (C) No, because the RTI Act only applies to state
meaning of information. It includes the fact that and central government monitored institutions.
information may include anything related to or (D) No, because private companies are exempt from
including Records, Documents, Memos, E-Mails, disclosing information under Section 4 of the
Opinions, Advice, Press releases, Circulars, Orders, RTI Act.
Logbooks, Contracts, Reports, Papers, Samples, etc.
Section 2 (h) – Defines the meaning of public 27. A citizen of Jammu and Kashmir files a request for
authority as any authority or bodies under the information under the RTI Act, 2005.Is the citizen of
Constitution of India, which include all state and Jammu and Kashmir entitled to exercise their right
central government monitored institutions as well. to information under the RTI Act?
The civil societies that are indirectly, or directly, (A) Yes, because the RTI Act extends to the whole
funded by the public funds are also included in. nation, including Jammu and Kashmir.
Section 2 (j) – ‘Right to Information’, as given in this (B) Yes, because the right to information is a
Act, means the Right to access any information that fundamental right of speech and expression.
is under the control of any public authority. The (C) No, because Jammu and Kashmir has its state-
Section further defines what else the public has level governance and is exempt from the RTI
access to through this Act. Act.
Section 4 – Deals with the maintainability and (D) No, because the RTI Act does not apply to civil
proactive disclosure of information. It states that the societies indirectly funded by public funds.
[28]
28. A citizen submits an RTI application seeking 30. A citizen approaches a lower court to seek redressal
information regarding an ongoing investigation by a regarding the denial of information by a public
public authority. Is the public authority obligated to authority under the RTI Act. Can the lower court
disclose information related to an ongoing entertain the citizen's suit or application in this
investigation under the RTI Act? regard?
(A) Yes, because Section 2(j) of the RTI Act grants (A) Yes, because the lower courts have jurisdiction
access to any information under the control of a over matters related to the RTI Act.
public authority. (B) Yes, because the citizen has a right to seek
(B) Yes, because the citizen has a right to know justice for the denial of information.
about the actions of public authorities. (C) No, because Section 23 of the RTI Act bars the
(C) No, because ongoing investigations fall under lower courts from entertaining such suits or
the exemption clause mentioned in Section 8 of applications.
the RTI Act. (D) No, because the writ jurisdiction of the Supreme
(D) No, because public authorities are not required Court and High Courts is not affected by
to disclose information under Section 6 of the Section 23 of the RTI Act.
RTI Act.

29. A citizen files an RTI application and receives no


response from the Public Information Officer (PIO)
within the prescribed time frame. Can penalty fees
be imposed on the PIO for the delay in providing
information?
(A) Yes, because Section 7 of the RTI Act
prescribes a time frame of 30 days for providing
information.
(B) Yes, because Section 20 of the RTI Act
provides penalty fees for delayed or distorted
information.
(C) No, because the RTI Act does not impose any
penalties for delayed responses by the PIO.
(D) No, because penalty fees can only be imposed
if the information provided is wrong or
distorted.

[29]
Quantitative Techniques
1. Simple interest received on ₹x at 10% rate after 2 final quantity of the water is what per cent of the final
years is ₹240 less than that received on ₹(x + 200) at quantity of the milk?
12% rate after 2 years. Find the simple interest (A) 10% (B) 15%
received on ₹(x – 600) at 15% rate after 3 years? (C) 20% (D) 25%
(A) ₹1620 (B) ₹1890 (E) None of these
(C) ₹2025 (D) 1440
(E) ₹1710 7. Anil got 24% and failed by 2x marks. While Ragu
got 36% and secured x marks more than the passing
2. A 5-digit number 13P7Q is divisible by 11 and P is marks. If Kumar scored 40% marks and got 100
1 more than Q. What is the smallest number to be marks, find the pass mark.
subtracted from the number to make it divisible by (A) 120 (B) 75
9? (C) 80 (D) 100
(A) 7 (B) 4 (E) 90
(C) 5 (D) 2
(E) 3 8. A dishonest seller marked up the price by 20% and
allows 25% discount on the item. While selling, he
3. B is 5 years older than A. The ratio of A’s age after uses a faulty weight of 750 grams instead of 1 kg,
4 years and B’s age five years ago is 6:5. If present then what is the overall profit percent earned by
age of C is 4/5th of the age of A, then find the seller?
average age of A & C. (A) 20% (B) 25%
(A) 12 (B) 14 (C) 10% (D) 15%
(C) 16 (D) 18 (D) None of these
(D) 15
9. P and Q together can complete half of the work in 32
4. If the ratio of income of Geeta and Sita is 5:4 and the days, while Q and R can do 60% of work in 36 days.
ratio of expenditure of Geeta and Sita is 7:3 P started the work alone and worked for 16 days,
respectively. If the savings of Geeta is Rs 800 more then Q alone worked for 52 days and R alone
than that of Sita and Expenditure of Geeta is Rs complete the remaining work in 52 days. Find the
1400. Find the total income of Geeta and Sita? time taken by P and R together to complete the work,
(A) 10800 (B) 14000 if P worked with 12.5% less efficiency?
(C) 14400 (D) 11200 (A) 60 days (B) 64 days
(D) None of these (C) 48 days (D) 54 days
(E) None of these
5. A person covers a distance from A to B and From B
to C and the ratio of the distance from A to B and 10. The swimmer's speed downstream is such that they
From B to C is 7:6. The average speed in this whole cover a distance of 84 km in 6 hours, while their
journey is 60 kmph and the total time taken in this speed upstream allows them to cover a distance of 36
journey is 2 hours 10 min, then find the distance from km in the same 6 hours. Determine the speed of the
A to B. stream.
(A) 50 (B) 80 (A) 4 km/hr (B) 3 km/hr
(C) 70 (D) 40 (C) 5 km/hr (D) 8 km/hr
(E) None of these (D) None of these

6. A milkman would have a 40 litres mixture of water


and milk in a ratio of 3:5. If he sold 16 litres of the
mixture and then 12 litres of milk added to the
mixture. Again he sold 20 litres of the mixture, and
then he added 8 litres of milk to the mixture. The

[30]
Answer Key
Section I 6. (A) 12. (D) 13. (D)
1. (A) 7. (B) 13. (D) 14. (C)
2. (B) 8. (C) 14. (A) 15. (B)
3. (D) 9. (B) 15. (A) 16. (B)
4. (A) 10. (C) 16. (C) 17. (B)
5. (D) 11. (A) 17. (B) 18. (D)
6. (B) 12. (A) 18. (C) 19. (B)
7. (A) 13. (C) 19 (B) 20. (A)
8. (D) 14. (A) 20 (D) 21. (D)
9. (B) 15. (B) 21. (A) 22. (A)
10. (B) 16. (A) 22. (C) 23. (D)
11. (B) 17. (C) 23. (D) 24. (D)
12. (B) 18. (C) 24. (B) 25. (A)
13. (B) 19. (A) 25. (B) 26. (A)
14. (B) 20. (C) 26. (A) 27. (C)
15. (A) 21. (C) 27. (C) 28. (C)
16. (A) 22. (A) 28. (B) 29. (B)
17. (B) 23. (A) 29. (A) 30. (C)
18. (B) 24. (B) 30. (C)
19. (A) 25. (C) Section V
20. (A) Section IV 1. (B)
21. (B) Section III 1. (D) 2. (D)
22. (B) 1. (B) 2. (C) 3. (D)
23. (C) 2. (C) 3. (A) 4. (C)
24. (B) 3. (B) 4. (D) 5. (C)
25. (C) 4. (C) 5. (D) 6. (C)
5. (C) 6. (D) 7. (C)
Section II 6. (C) 7. (A) 8. (A)
1. (A) 7. (B) 8. (B) 9. (A)
2. (B) 8. (C) 9. (A) 10. (A)
3. (B) 9. (B) 10. (C)
4. (C) 10. (D) 11. (B)
5. (B) 11. (A) 12. (D)

[31]
Hints & Solutions
Section I However, the passage mentions that besides guidance
PASSAGE I and data on cost-cutting measures, the Big Tech
1. Correct Option A earnings season was dominated by AI detail. This
(H & S) suggests that the focus of the earnings reports was
Option A is the correct answer. The passage states primarily on AI-related information, updates, and
that Warren Buffett and Charlie Munger expressed strategies. Therefore, the most accurate option is D)
skepticism about AI and mentioned that they believe "The guidance for managing an economic downturn."
traditional intelligence works well. This aligns with This aligns with the passage's emphasis on the
their viewpoint that AI is overhyped. importance of guidance and the tech giants' awareness
Option B is incorrect. The passage does not mention of the potential returns when AI starts to change the
any expression of optimism from Buffett and way businesses work.
Munger regarding the potential of AI in the tech
industry. 3. Correct Option D
Option C is incorrect. The passage does not discuss (H & S)
the financial performance of tech giants in the AI The passage mentions that the Big Tech earnings
sector in relation to Buffett and Munger's skepticism. season was dominated by AI detail. This means that
It only mentions that tech giants posted robust AI-related discussions, information, and updates were
financial performance reports, but their skepticism is the main focus during the earnings season. The
separate from that. passage highlights that tech giants are highly
Option D is incorrect. While the passage suggests interested in AI and believe it to be the future.
including AI stocks in most investors' portfolios, this Therefore, it is reasonable to conclude that AI was the
viewpoint is attributed to Nigel Green, the CEO of central topic of discussion and analysis during the Big
deVere Group, and not Warren Buffett and Charlie Tech earnings season. This aligns with the statement
Munger. They express skepticism about AI, as made by Nigel Green, the CEO and founder of deVere
mentioned in option A. Group, who emphasized the tech giants' relentless
focus on AI. Thus, option d) "AI detail" is the correct
2. Correct Option D answer.
(H & S) The passage does not provide information to support
The passage states that the Big Tech earnings season options a) "Corporate guidance," b) "Data on cost-
was dominated by AI detail. While the passage does cutting measures," or c) "Robust financial
mention the robust financial performance of tech performance reports" as the dominant topic during the
giants during the first-quarter earnings, it does not Big Tech earnings season. While these aspects might
specifically highlight that this was the focus of the have been covered during the earnings season, the
reports. Similarly, while the passage discusses the passage specifically highlights the significance of AI-
potential returns from investing in AI stocks, it does related discussions.
not state that this was the primary focus of the
earnings reports. The passage also does not provide 4. Correct Option A
information to support option B) "The impact of AI (H & S)
on society and global business" as the focus of the According to the passage, the tech giants mentioned
earnings reports. in the article have shown a strong interest in AI and

[32]
believe it to be the future. The passage also mentions 2. As stated in the passage, the Telcom Regulatory
that companies with substantial AI interests are likely Authority of India (Trai) is considering revising
the current centralised audit process to a circle-
to benefit from the growth of the industry, which
wise approach for the audit of billing and metering
could result in significant rewards for early investors. practices of telcos. This aligns with the
This suggests that AI has the potential to create information provided, making option B the correct
answer.
lucrative opportunities for investors. Therefore,
3. The passage does not mention the Telcom
option a) "It could lead to enormous rewards for early Regulatory Authority of India (Trai) considering
investors" is the correct answer. eliminating the audit of billing and metering
practices for telcos. Therefore, this option is not
The passage does not provide information to support
the correct answer.
options b) "It could result in substantial losses for 4. While the passage mentions that telecom operators
investors," c) "It could cause a decline in the tech are concerned about the potential change in the
industry," or d) "It could result in a decrease in audit process and its impact on their regulatory
burden, it does not directly state that the Telcom
employment opportunities." These options are not Regulatory Authority of India (Trai) is considering
directly addressed in the passage and do not align with increasing the regulatory burden on telecom
the positive outlook on AI expressed in the passage. operators. Therefore, this option is not the correct
answer.

5. Correct Option D 7. Correct Option A


(H & S) (H & S)
1. As mentioned in the passage, telecom operators
In the passage, it is mentioned that the AI chatbot
are concerned about the potential increase in their
ChatGPT became wildly popular in just a matter of regulatory burden due to the Trai's proposal of
weeks and had 100 million monthly active users conducting micro-audits at the circle level. This
option correctly identifies the concern raised by
within two months of its launch. The passage also
telecom operators, making it the correct answer.
compares its rapid growth to social media platforms 2. The passage mentions that the regulator is
like TikTok or Instagram. Based on this information, considering a shift from the centralized audit
we can infer that the purpose of ChatGPT was not to process to conducting audits of billing and
metering at the circle level. However, this option
provide financial advice to investors (option a), help
does not capture the specific concerns expressed
businesses work more efficiently (option b), or serve by telecom operators, making it an incorrect
as a social media platform (option c). Instead, the choice.
3. While the passage mentions that the regulator may
passage suggests that the primary purpose of
demand the deployment of new technological
ChatGPT was to demonstrate the potential of AI. Its mechanisms, it does not explicitly state that
popularity and adoption by a large user base highlight telecom operators are concerned about the
excessive cost associated with it. Therefore, this
the success and effectiveness of AI technology in the
option does not reflect the concerns expressed by
form of a chatbot. Therefore, option d) "To the operators.
demonstrate the potential of AI" is the correct answer. 4. The passage mentions that the Trai plans to
conduct a probe of past offers by all operators to
PASSAGE II determine any violations. However, the passage
does not indicate that telecom operators are
6. Correct Option B specifically concerned about facing penalties for
(H & S) past instances of predatory pricing and unlimited
1. The passage does not mention the Telcom data offers. Thus, this option does not reflect the
Regulatory Authority of India (Trai) considering concerns raised by the operators.
conducting micro-audits at the central level
instead of the circle level. Therefore, this option is 8. Correct Option D
not the correct answer. (H & S)
[33]
1. The passage does not explicitly mention that light or actions of telecom operators in resolving issues.
touch regulation supports a more centralized audit Therefore, this option is not the correct answer.
process, reducing the burden on telecom operators.
Therefore, this option does not strengthen the 10. Correct Option B
suggested approach for resolving issues, and it is (H & S)
not the correct answer. 1. Option A does not contain any grammatical errors.
2. While the passage mentions that the regulator may It is a grammatically correct sentence.
demand the deployment of new technological 2. Option B contains a grammatical error. The phrase
mechanisms, it does not explicitly state that light "audit of billing and metering of telcos" should be
touch regulation promotes their use to streamline "audit of billing and metering by telcos" for correct
billing and metering audits. Hence, this option syntax. Therefore, this option is the correct
does not provide a strengthening argument and is answer.
not the correct answer. 3. Option C does not contain any grammatical errors.
3. The passage does not indicate that light touch It is a grammatically correct sentence.
regulation favors conducting micro-audits at the 4. Option D does not contain any grammatical errors.
circle level. Instead, it mentions that telecom It is a grammatically correct sentence.
operators view the proposed micro-audits as
superfluous and burdensome. Therefore, this PASSAGE III
option does not strengthen the suggested approach
and is not the correct answer. 11. Correct Option B
4. The passage suggests that light touch regulation (H & S)
1. Option A does not weaken the argument for
has become a hallmark of Trai. It also suggests that
calling a meeting of all the operators to resolve allowing a democratically elected government to
issues across the table would be a better approach. serve its constituents. It actually aligns with the
This aligns with the concept of open dialogue and
argument by emphasizing the importance of
resolving issues through discussions among
stakeholders. Thus, option D strengthens the cooperative federalism and balanced power-
suggested approach for resolving issues and is the sharing. Therefore, this option is not the correct
correct answer.
answer.

9. Correct Option B 2. Option B weakens the argument by suggesting that


(H & S) the working relationship between the chief
1. The word "procrastination" refers to the act of minister's office and the lieutenant governor's
delaying or postponing something. However, the
passage does not suggest that Trai favors a office had been effective for several decades,
procrastinating approach in resolving issues with indicating that shared power can work well. This
telecom operators. Therefore, this option is not the weakens the argument for allowing exclusive
correct answer.
control by the elected government. Thus, this
2. The word "facilitation" refers to the act of making
a process easier or smoother. The passage suggests option is the correct answer.
that Trai should call a meeting of all operators to 3. Option C strengthens the argument by highlighting
resolve issues, which aligns with the idea of
the negative consequences of the power struggle
facilitating discussions. Thus, option B represents
the correct answer. between the elected government and the lieutenant
3. The word "denunciation" means publicly governor, such as annulled appointments, policy
condemning or criticizing someone or something.
delays, and communication breakdowns. This
The passage does not indicate that Trai favors a
denunciatory approach in resolving issues with supports the need for allowing a democratically
telecom operators. Hence, this option is not the elected government to serve its constituents
correct answer. effectively. Therefore, this option is not the correct
4. The word "invalidation" means declaring
answer.
something as invalid or nullifying it. The passage
does not mention Trai invalidating any decisions

[34]
4. Option D strengthens the argument by not align with the information provided and is not
emphasizing the importance of accountable and the correct answer.
responsive civil service, which suggests that
elected representatives alone may not ensure 13. Correct Option B
efficient governance. This aligns with the (H & S)
argument for allowing a democratically elected 1. The passage does not mention any collaboration or
government. Thus, this option is not the correct effective resolution of the power struggle between
answer. the Aam Aadmi Party (AAP) government and the
Bharatiya Janata Party (BJP). It focuses on the
12. Correct Option B negative consequences of the power struggle and
(H & S) the Supreme Court's role in resolving the issue.
1. The passage does not explicitly state that the Therefore, this option cannot be inferred from the
lieutenant governor's office has historically held passage and is not the correct answer.
more power and authority than the elected 2. The passage states that the power struggle between
government in Delhi. It mentions a power struggle the Aam Aadmi Party (AAP) government and the
between the two, but does not provide information Bharatiya Janata Party (BJP) has led to ineffective
about the comparative power dynamics. governance in Delhi, with annulled appointments,
Therefore, this option cannot be inferred from the policy delays, and communication breakdowns.
passage and is not the correct answer. This supports the inference that the power struggle
2. The passage mentions that the power struggle has been the primary cause of ineffective
between the elected government and the lieutenant governance. Thus, this option is the correct
governor's office has led to ineffective answer.
governance, with annulled appointments, policy 3. The passage does not provide any information
delays, and communication breakdowns. This about the Bharatiya Janata Party (BJP) supporting
supports the inference that the power struggle has the Aam Aadmi Party (AAP) government in
negatively affected governance in Delhi. Thus, asserting control over assigned bureaucrats. This
this option is the correct answer. option goes beyond the information provided and
3. The passage does not state that the recent Supreme cannot be inferred, so it is not the correct answer.
Court judgment has resolved the power struggle 4. The passage does not mention any mutual
between the elected government and the lieutenant agreement or power-sharing arrangement between
governor's office. It mentions that the judgment the Aam Aadmi Party (AAP) government and the
confirmed the government's control over assigned Bharatiya Janata Party (BJP). It focuses on the
bureaucrats, but does not indicate a resolution of power struggle and the Supreme Court's
the power struggle. Therefore, this option goes intervention. Therefore, this option goes beyond
beyond the information provided and is not the the information provided and is not the correct
correct answer. answer.
4. The passage mentions that amendments to the
Government of National Capital Territory of Delhi 14. Correct Option B
Act in 2021 led to the expansion of the lieutenant (H & S)
governor's purview, rather than the elected 1. The passage does not mention the resolution of the
government's purview. Therefore, this option does power struggle between the Aam Aadmi Party

[35]
(AAP) government and the Bharatiya Janata Party government has the legal authority over those
(BJP) through the intervention of the Supreme departments. Therefore, option A accurately
Court. It discusses the Supreme Court's judgment, represents the meaning of "purview" in the
but it does not provide information about the passage and is the correct answer.
resolution of the power struggle. Therefore, this 2. The term "purview" does not relate to a state of
option goes beyond the information provided and disagreement or conflict between two parties. It is
is not the correct answer. focused on the authority or control over a specific
2. The passage explicitly states that the power area. Therefore, this option does not accurately
struggle between the Aam Aadmi Party (AAP) represent the meaning of "purview" in the passage
government and the Bharatiya Janata Party (BJP) and is not the correct answer.
has resulted in ineffective governance, with 3. The term "purview" does not refer to the act of
annulled appointments, policy delays, and intervening or mediating in a dispute or conflict. It
communication breakdowns. This supports the is more about the extent of authority or control.
conclusion that the power struggle has negatively Therefore, this option does not accurately
impacted the effectiveness of governance in Delhi. represent the meaning of "purview" in the passage
Thus, this option is the correct answer. and is not the correct answer.
3. The passage does not provide information to 4. The term "purview" does not pertain to the process
support the conclusion that the power struggle of reaching a mutual agreement or understanding.
between the Aam Aadmi Party (AAP) government It is more about the scope of authority or control.
and the Bharatiya Janata Party (BJP) is a common Therefore, this option does not accurately
phenomenon observed in democratic governments represent the meaning of "purview" in the passage
worldwide. It focuses on the specific power and is not the correct answer.
struggle in Delhi. Therefore, this option goes
beyond the information provided and is not the PASSAGE IV
correct answer. 16. Correct Option A
4. The passage does not mention collaborative efforts (H & S)
between the Aam Aadmi Party (AAP) government 1. The passage highlights the arrest of former prime
and the Bharatiya Janata Party (BJP) in improving minister Imran Khan, the subsequent protests
governance in Delhi. It emphasizes the negative against the Pakistani military's interference, and
consequences of the power struggle. Therefore, the growing rift between Khan's party and the
this option goes beyond the information provided military. These events indicate a significant point
and is not the correct answer. of contention between Khan's Pakistan Tehreek-e-
Insaf (PTI) and the military. This aligns with the
15. Correct Option A main point of the passage, making option A the
(H & S) correct answer.
1. The term "purview" refers to the range or scope of 2. While option B mentions Imran Khan's rise to
one's authority or control over a specific area or power with the support of the military and strained
domain. In the passage, it is mentioned that the relations due to his attempts to assert control over
Aam Aadmi Party (AAP) government controlled military appointments and criticize the military, it
bureaucrats assigned to departments that are does not encompass the wider context of the arrest,
legally within its purview. This indicates that the protests, and the current state of affairs. Therefore,

[36]
this option does not accurately represent the main criticized them. Therefore, option B
point of the passage and is not the correct answer. accurately represents an inference from the
3. Option C focuses on the challenges faced by the passage and is the correct answer.
Pakistani military in maintaining a hybrid regime 3. While the passage mentions that Imran Khan
and conflicts with political figures like Imran was propelled to power with the support of
Khan, but it does not address the specific events of the Pakistani military, it does not provide
the arrest, protests, and the growing rift between information on whether he relied on their
Khan's party and the military. Therefore, this support to maintain his position as prime
option does not accurately represent the main point minister. Therefore, this option does not
of the passage and is not the correct answer. accurately represent an inference from the
4. Option D mentions General Asim Munir's role and passage and is not the correct answer.
challenges in balancing the military's interests and 4. The passage states that relations between
influence amidst economic challenges and Imran Khan and the Pakistani military
escalating terror strikes. While this is part of the nosedived with his repeated public potshots at
broader context, it does not capture the specific the military and allegations against them.
events of the arrest, protests, and the growing rift This indicates a strained relationship rather
between Khan's party and the military. Therefore, than a strong alliance. Therefore, this option
this option does not accurately represent the main does not accurately represent an inference
point of the passage and is not the correct answer. from the passage and is not the correct
answer.
17. Correct Option B Correct Answer: B. Imran Khan's attempts to assert
(H & S) control over military appointments and criticize the
1. The passage does not provide any evidence or military led to strained relations with the Pakistani
information indicating that Imran Khan had a military.
harmonious relationship with the Pakistani
military throughout his tenure as prime 18. Correct Option B
minister. In fact, it states that relations (H & S)
between Khan and the military strained as he 1. The passage mentions that several Pakistan Army
sought to interfere in military appointments chiefs, including General Ashfaq Pervez Kayani,
and publicly criticized the military. have spoken of steering clear of politics. However,
Therefore, this option does not accurately there is no direct evidence in the passage to
represent an inference from the passage and support the inference that the Pakistani military
is not the correct answer. has maintained a neutral stance in politics since
2. The passage mentions that Imran Khan's Kayani's tenure. Therefore, this option does not
attempts to assert control over military accurately represent an inference from the passage
appointments and criticize the military and is not the correct answer.
resulted in strained relations with the 2. The passage suggests that the Pakistani military
Pakistani military. This is supported by the has shifted to a role where it can manage the
passage's statement that the military became government by remote control, indicating that it
wary of Khan as he sought to interfere in the continues to intervene in politics but with a more
posting of senior generals and publicly subtle approach. This is supported by the

[37]
statement that the military has interests and relationship between the military and elected
influence over Pakistan's destiny at stake. representatives. Therefore, option B does not
Therefore, option B accurately represents an weaken the argument and is not the correct answer.
inference from the passage and is the correct 3. Option C neither weakens nor strengthens the
answer. argument. It simply restates a previously
3. The passage does not provide evidence or mentioned factor without providing any new
information indicating that the Pakistani military's information or perspective. Therefore, option C
influence in politics has diminished or that it has does not weaken the argument and is not the
led to a more stable government. Therefore, this correct answer.
option does not accurately represent an inference 4. Option D strengthens the argument by suggesting
from the passage and is not the correct answer. an ongoing collaboration between the military and
4. The passage mentions that a hybrid regime is elected representatives, as evidenced by the
unworkable and that the military's deepening military's go-ahead for the parliamentary vote of
unpopularity and experience with Imran Khan confidence that ousted Imran Khan from power.
indicate this. However, there is no evidence in the This supports the idea that the arrest of Imran
passage to suggest that the Pakistani military is Khan signifies a significant shift in the relationship
actively seeking to establish a hybrid regime by between the military and elected representatives.
managing the government remotely. Therefore, Therefore, option D does not weaken the argument
this option does not accurately represent an and is not the correct answer.
inference from the passage and is not the correct Correct Answer: A. The protests triggered by the
answer. arrest of Imran Khan were primarily driven by his
Correct Answer: B. The Pakistani military continues supporters rather than being representative of a
to actively intervene in politics, but with a more subtle broader sentiment against the Pakistani military.
approach.
20. Correct Option A
19. Correct Option A (H & S)
(H & S) 1. Option A correctly defines the term "potshots" as
1. Option A weakens the argument by suggesting that used in the passage. "Potshots" refers to direct
the protests triggered by Imran Khan's arrest were criticisms or attacks against the military. This is
driven by his supporters rather than being supported by the context in the passage, where it
indicative of a broader sentiment against the is mentioned that Imran Khan repeatedly made
Pakistani military. This implies that the arrest may public potshots at the military. Therefore, option
not necessarily signify a significant shift in the A is the correct answer.
relationship between the military and elected 2. Option B incorrectly defines the term "potshots"
representatives. Therefore, option A is the correct as casual and playful remarks aimed at diffusing
answer. tension. This definition does not align with the
2. Option B strengthens the argument by indicating context of the passage, where the term is used to
the military's direct involvement in the arrest of describe Imran Khan's criticisms against the
Imran Khan through a paramilitary force led by military. Therefore, option B is not the correct
Pakistan Army officers. This supports the idea that answer.
the arrest signifies a significant shift in the

[38]
3. Option C incorrectly defines the term "potshots" their support for the war in Yemen, which is why
as secret negotiations and dealings to retain this is the correct answer.
political power. This definition is unrelated to the 3. The passage does not mention the peace deal
context of the passage, which focuses on the imposing sanctions on the Houthi rebels.
strained relationship between Imran Khan and the 4. The peace deal does not provide for the
military. Therefore, option C is not the correct deployment of UN peacekeeping forces in Yemen
answer. as per the passage.
4. Option D incorrectly defines the term "potshots"
as intense and confrontational protests against the 23. Correct Option C
government. This definition is not applicable to (H & S)
the context of the passage, where the term is used 1. Although China's role in brokering the peace deal
to describe Imran Khan's criticisms against the between Saudi Arabia and Iran is mentioned, it is
military. Therefore, option D is not the correct not the main theme of the passage.
answer. 2. While the political history of Yemen and the peace
Correct Answer: A. Direct criticisms or attacks deal are discussed, this option does not
against the military encapsulate the entire scope of the passage, which
also delves into the internal and external conflicts
PASSAGE V in Yemen.
21. Correct Option B 3. This option correctly summarizes the main idea of
(H & S) the passage, as it encompasses all the significant
1. The passage does not mention an invasion of Saudi points: the internal and external conflicts in
Arabia triggering the internal conflict in Yemen. Yemen, their historical roots, and the recent peace
2. As the passage states, the roots of the internal deal that offers hope for their resolution.
conflict in Yemen go back to the unification of Therefore, this is the correct answer.
North and South Yemen in 1990, making this the 4. Although the operation of the Houthi Supreme
correct answer. Political Council in Yemen is mentioned, it is not
3. The airstrikes launched by Saudi Arabia and the the main theme of the passage. This option is too
US in 2015 were a response to the armed conflict narrow to represent the main idea of the entire
in Yemen, not a trigger for the internal conflict. passage.
4. The British withdrawal from South Yemen Hence Option C is the correct answer.
occurred in 1967, but it wasn't the trigger for the
internal conflict in Yemen as per the details in the 24. Correct Option B
passage. (H & S)
1. The passage does not suggest a cooperative
22. Correct Option B relationship between North Yemen and South
(H & S) Yemen before their unification, making this option
1. There's no mention in the passage of the peace deal incorrect.
allowing for China's military intervention in 2. As mentioned in the passage, North Yemen and
Yemen. South Yemen were divided along a north-south
2. As per the peace deal mentioned in the passage, divide, indicating a history of division, which is
both Saudi Arabia and Iran have agreed to end the correct answer.

[39]
3. The passage does not mention a symbiotic Section II
relationship between North Yemen and South PASSAGE I
Yemen. 1. Correct Option A
4. The passage does not indicate a collaborative (H & S)
relationship between North Yemen and South Correct Option: A) India's economic structure has
Yemen. undergone significant changes over the past four
decades, with the services sector becoming dominant.
25. Correct Option A Reasoning: This option accurately captures the main
(H & S) idea of the passage, which discusses the shift in India's
1. The historical conflict between North and South economic structure over time, highlighting the decline
Yemen, mentioned in the passage, provides in the agriculture sector's share and the rise of the
important context to the internal conflict in services sector as the dominant contributor to GDP.
Yemen. However, it does not directly undermine Incorrect Options:
the hope for a lasting peace in Yemen as per the B) India's economic growth has accelerated due to the
details provided in the passage. Therefore, this dominance of the rapidly growing services sector.
option is not the correct answer. This option misinterprets the main idea by stating that
2. The passage does not mention the lack of economic growth has accelerated solely due to the
involvement of the United States in the peace dominance of the services sector. The passage
process. While the passage mentions the US's past discusses the structural changes in India's economy
involvement in launching airstrikes in Yemen, it but does not directly mention the impact on economic
does not provide information about the current growth.
peace process or the role of the United States. C) The shift in weighting among sectors has resulted
Thus, this option cannot be considered as the in India's potential for annual growth reaching at least
correct answer. 7 percent.
3. According to the passage, the ongoing support of This option goes beyond the scope of the passage.
external powers to the warring factions can While the passage mentions factors that could
undermine the hope for a lasting peace in Yemen. contribute to India's potential growth rate, it does not
The peace deal between Saudi Arabia and Iran explicitly state that the shift in sector weighting has
may offer hope, but if external powers continue to led to a specific growth rate of 7 percent.
provide support to the warring factions, it can D) The International Monetary Fund (IMF) has
prolong the conflict and hinder the peace process. expressed concerns about India's growth potential due
This aligns with the information given in the to the impact of the pandemic and other factors.
passage, making option C the correct answer. This option does not accurately represent the main
4. The passage does not mention the reluctance of idea of the passage. Although the passage mentions
Houthi rebels to participate in peace talks. While the IMF's assessment of India's growth rate, it is not
it states that there have been talks between Saudi the central focus or main idea of the passage.
and Omani delegations with Houthi officials, it
does not provide information on the willingness or 2. Correct Option B
reluctance of the Houthi rebels to engage in peace (H & S)
talks. Therefore, this option cannot be considered Correct Option: B) The statement is moderately
as the correct answer. supported, as the passage mentions some factors that

[40]
could contribute to India's potential growth but does Reasoning: The passage mentions the rapid spread of
not provide comprehensive evidence. education and digitization as factors that could
Reasoning: The passage discusses various factors improve productivity. Therefore, it can be inferred
such as changes in economic structure, increased life that these factors have positively impacted
expectancy, education, investment in fixed capital, productivity and economic growth in India.
and digitization that could contribute to India's Incorrect Options:
potential growth. However, it does not provide A) The IMF's assessment of India's growth rate is
comprehensive evidence or a definitive conclusion widely accepted by the government and domestic
regarding the exact degree of India's potential growth. commentators.
Incorrect Options: This option introduces information that is not
A) The statement is highly supported, as the passage mentioned in the passage. The passage does not
presents various factors that suggest India's potential indicate whether the government or domestic
growth rate should be at least 7 percent. commentators accept the IMF's assessment.
This option goes beyond the information provided in C) The average life expectancy of Indians has
the passage. While the passage mentions factors that increased significantly, leading to higher productivity
could contribute to India's potential growth rate, it and economic growth.
does not specifically state that the growth rate should This option correctly states one of the factors
be at least 7 percent. mentioned in the passage. However, the passage does
C) The statement is minimally supported, as the not explicitly state that the increase in life expectancy
passage discusses changes in India's economy but has directly led to higher productivity and economic
does not directly address its potential growth rate. growth.
This option underestimates the level of support D) The pandemic has had a significant negative
provided in the passage. The passage discusses impact on India's potential for economic growth.
several factors that could contribute to India's This option goes beyond the information provided in
potential growth rate, although it does not provide a the passage. While the passage mentions the impact
specific growth rate. of the pandemic on India's growth, it does not specify
D) The statement is unsupported, as the passage the magnitude of the impact or whether it has
primarily focuses on the shift in India's economic significantly affected India's potential for economic
structure and the IMF's assessment of its growth rate. growth.
This option misrepresents the information in the
passage. The passage does mention factors 4. Correct Option C
contributing to India's potential growth rate but does (H & S)
not solely focus on the IMF's assessment or lack Correct Option: C) India's educational quality has
thereof. improved significantly, leading to a highly skilled
workforce and increased productivity.
3. Correct Option B Reasoning: This option strengthens the argument by
(H & S) highlighting the positive impact of improved
Correct Option: B) India's rapid spread of education educational quality on productivity and economic
and digitization has positively impacted productivity growth. The passage mentions the rapid spread of
and economic growth. education, but it also acknowledges that educational
quality is an issue.

[41]
Incorrect Options: A) The IMF's assessment of India's growth rate aligns
A) The services sector has consistently outperformed with the growth assessments of the government and
other sectors in terms of growth rate over the past four the Reserve Bank.
decades. This option is unrelated to weakening the argument. It
This option introduces information that is not directly introduces information about the alignment of growth
relevant to the argument. While the passage discusses assessments without addressing the potential
the dominance of the services sector, it does not hindrances to India's growth.
specifically mention its growth rate relative to other C) The services sector, despite its dominance, has
sectors. shown signs of slowdown and reduced growth
B) The rate of investment in fixed capital has potential.
remained stable, indicating a favorable environment This option introduces information that is not
for economic growth. mentioned in the passage. The passage does not
This option introduces information that is not directly discuss any signs of slowdown or reduced growth
related to the argument. The passage mentions an potential in the services sector.
increase in the rate of investment in fixed capital, but D) The pandemic has had a minimal impact on India's
it does not explicitly discuss the stability of this rate potential for economic growth, allowing for a rapid
or its direct impact on economic growth. recovery.
D) The government and the Reserve Bank of India This option strengthens rather than weakens the
have projected higher growth rates for the current year argument by suggesting that the pandemic had a
compared to the IMF's estimate. minimal impact on India's potential for economic
This option introduces information that is not growth. However, the passage does not provide
mentioned in the passage. The passage does not information about the extent of the pandemic's impact
provide information about the projections of the or the speed of recovery.
government or the Reserve Bank of India regarding
growth rates. PASSAGE II
6. Correct Option A
5. Correct Option B (H & S)
(H & S) Correct Option: A) Studies have shown that moderate
Correct Option: B) India's economy has faced smartphone use has no negative impact on mental
significant challenges, such as income inequality and health.
a lack of inclusive growth. Reasoning: This option weakens the argument by
Reasoning: This option weakens the argument by suggesting that moderate smartphone use does not
introducing the idea that India's economy has faced have a negative impact on mental health. It implies
challenges that could hinder its potential for growth. that the negative effects mentioned in the passage may
The mention of income inequality and a lack of be the result of excessive or addictive smartphone use
inclusive growth suggests that there are obstacles to rather than smartphone use in general.
achieving high and inclusive economic growth in Incorrect Options:
India. B) Smartphone addiction is not a widespread issue
Incorrect Options: and only affects a small percentage of the population.
This option introduces new information that is not
mentioned in the passage. The passage discusses the

[42]
deleterious effects of smartphone addiction without in the passage states that the deleterious effects of
providing information about the prevalence of the smartphones are more pronounced on the mental
issue. health of women.
C) The use of smartphones can actually improve D) Cyberbullying of women is a rising concern
cognitive function and self-esteem in individuals. associated with smartphone use.
This option introduces new information that The passage mentions that cases of cyberbullying of
contradicts the argument presented in the passage. women are on the rise, supporting this statement.
The passage suggests that heavy smartphone use leads Therefore, it is not false based on the information
to impaired cognitive function and low self-esteem, provided.
not improvement.
D) The health issues attributed to smartphone 8. Correct Option C
addiction can be easily mitigated through regular (H & S)
exercise and healthy lifestyle choices. Correct Option: C) The content on smartphones often
This option introduces a potential solution to mitigate perpetuates misogyny and gender discrimination.
the health issues associated with smartphone Reasoning: This inference is supported by the
addiction but does not directly weaken the argument. passage, which mentions that smartphones
It does not challenge the claim that heavy smartphone disseminate content coated with misogyny and gender
use leads to addiction and health issues. discrimination. The passage suggests that women are
at a greater risk of suffering from mental health issues
7. Correct Option B due to exposure to such content.
(H & S) Incorrect Options:
Correct Option: B) Men are more likely to experience A) Women are more susceptible to smartphone
suicidal thoughts and hallucinations as a result of addiction compared to men.
smartphone addiction. The passage does not provide information about the
Reasoning: This statement contradicts the information susceptibility of women and men to smartphone
provided in the passage. The passage explicitly states addiction. Therefore, this inference goes beyond the
that the number of women affected by mental health information provided.
issues related to smartphone addiction was higher B) The use of smartphones at an early age is directly
than the number of men. linked to aggressive behavior.
Incorrect Options: While the passage mentions that those who got their
A) The age at which a person gets their first first smartphones at the age of six demonstrated
smartphone has no impact on their mental well-being. aggressive behavior, it does not establish a direct link
The passage provides evidence from the report by between the use of smartphones at an early age and
Sapien Labs that connects the age of first smartphone aggressive behavior. Therefore, this inference is not
ownership to mental well-being outcomes. Therefore, supported by the passage.
this statement is not false based on the information D) The prevalence of smartphone usage among Indian
provided. children is higher than the global average.
C) The mental health effects of smartphone addiction The passage mentions that smartphone usage among
are more pronounced in women compared to men. Indian children is as high as 83%, but it does not
This statement aligns with the information provided provide information about the global average.
in the passage. The report by Sapien Labs mentioned

[43]
Therefore, this inference goes beyond the information but presents them as potential measures. Therefore,
provided. this option misrepresents the argument's assumption.

9. Correct Option B 10. Correct Option C


(H & S) (H & S)
Correct Option: B) The argument does not consider Correct Option: C) Create a support structure to
other factors that could contribute to mental health provide timely counseling and assistance to
issues, such as underlying mental health conditions or individuals experiencing mental health issues related
socioeconomic factors. to smartphone addiction.
Reasoning: This option highlights a flaw in the Reasoning: This option proposes a course of action
argument by pointing out that the argument does not that addresses the concern of mental health issues
consider other factors that could contribute to mental related to smartphone addiction. It suggests the
health issues. The passage solely focuses on the creation of a support structure to offer counseling and
negative effects of smartphone use without assistance to individuals experiencing such issues.
acknowledging the potential influence of underlying Incorrect Options:
mental health conditions or socioeconomic factors. A) Implement strict regulations and surveillance
Incorrect Options: measures to monitor smartphone usage among
A) The passage fails to provide any evidence or children.
research studies to support the claim of smartphones This option suggests a course of action focused on
causing addiction and health issues. strict regulations and surveillance measures without
The passage mentions a report by Sapien Labs that addressing the need for support and counseling. It
connects the age of first smartphone ownership to goes beyond the scope of the passage.
mental well-being outcomes. Therefore, the passage B) Conduct further research to determine the exact
does provide evidence to support the claim. impact of smartphone use on mental health in
C) The passage relies solely on data from a single different demographic groups.
report by Sapien Labs without considering other While further research may be beneficial, it does not
studies or perspectives on the topic. directly address the immediate need for support and
The passage does mention a report by Sapien Labs, counseling for individuals experiencing mental health
but it does not claim to rely solely on this report. issues related to smartphone addiction. Therefore, this
Therefore, this option exaggerates the flaw in the option is not the most appropriate course of action
argument. based on the information provided.
D) The argument assumes that regulating and D) Educate parents and teachers on the importance of
restricting smartphone use among children and gatekeeping and responsible smartphone use among
sensitizing women to harmful content will effectively children.
prevent mental health issues. While education of parents and teachers is mentioned
The passage suggests the need for a protective edifice in the passage, it does not address the need for a
and gatekeeping when it comes to permitting children support structure to provide counseling and assistance
to use smartphones and sensitizing women to harmful to individuals experiencing mental health issues
content. The argument does not assume that these related to smartphone addiction. Therefore, this
actions will effectively prevent mental health issues option does not fully capture the most appropriate
course of action.

[44]
PASSAGE III Correct option: a) Kiren Rijiju's criticism of the
11. Correct Option A collegium system of judges' appointments had no
(H & S) impact on the judiciary's stance.
Correct option: a) Kiren Rijiju voluntarily requested a Reasoning:
transfer to the earth sciences ministry due to his Based on the passage, it is mentioned that Kiren
personal interest in the subject. Rijiju's aggression and confrontational rhetoric did
Reasoning: not dent the judiciary's stand or ruffle the dignity of
If it is true that Kiren Rijiju requested the transfer its senior members. Therefore, it can be inferred that
himself due to his personal interest in earth sciences, his criticism had no impact on the judiciary's stance.
it would indicate that the government's motive behind Other options:
the change in portfolio was not related to the judiciary b) Arjun Ram Meghwal's appointment as the new law
or a desire for a smoother relationship with the minister was primarily influenced by his seniority
Supreme Court. This would weaken the speculation within the Bharatiya Janata Party.
about the government's intention. This option is not necessarily true based on the
Other options: information provided in the passage. The passage
b) The Supreme Court recently praised Kiren Rijiju does not provide any specific information about the
for his efforts in advocating for judicial reforms. criteria for Arjun Ram Meghwal's appointment.
This option does not necessarily weaken the c) Kiren Rijiju's confrontational rhetoric towards
speculation as it could be argued that the praise from retired judges was openly supported by the vice-
the Supreme Court was part of the government's president.
strategy to improve its relationship with the judiciary. This option is not mentioned in the passage. The
c) The government has maintained a cordial passage only mentions that Kiren Rijiju's remarks ran
relationship with the Supreme Court throughout parallel to the vice-president's, but it does not
Narendra Modi's tenure as Prime Minister. explicitly state that the vice-president openly
This option does not weaken the speculation because supported his confrontational rhetoric.
it does not provide any specific information about d) Kiren Rijiju's transfer to the earth sciences ministry
Kiren Rijiju's change in portfolio or the government's was a strategic move by the government to strengthen
motive behind it. its relationship with the Supreme Court.
d) The Lok Sabha elections have been postponed, This option is not necessarily true based on the
eliminating the need for the government to establish a information provided in the passage. While the
smoother relationship with the judiciary at present. passage speculates about the government's motive
This option does not weaken the speculation as the behind the transfer, it does not provide definitive
postponement of elections does not necessarily evidence that the transfer was a strategic move to
eliminate the government's motive to improve its strengthen the relationship with the Supreme Court.
relationship with the judiciary. There could be other
reasons for the change in portfolio. 13. Correct Option C
(H & S)
12. Correct Option A Correct option: c) The government's high command
(H & S) disapproved of Kiren Rijiju's increasingly aggressive
stance.
Reasoning:

[45]
The passage mentions that Kiren Rijiju's with the Supreme Court. However, this is presented
confrontational rhetoric probably had his superiors' as speculation and not confirmed information. The
(government's) approval. However, it also states that author assumes that Kiren Rijiju's aggressive remarks
the sudden change of portfolio without explanation were the primary reason for his transfer, which may
suggests that the government may be looking for a not necessarily be the case.
less abrasive relationship with the Supreme Court. Other options:
This implies that the government's high command b) The author assumes that the government's motive
disapproved of Kiren Rijiju's aggressive approach behind Kiren Rijiju's change in portfolio is solely
towards the judiciary. related to its relationship with the Supreme Court.
Other options: This assumption is based on the speculation presented
a) Kiren Rijiju's aggressive remarks were welcomed in the passage, but it is not explicitly stated as the only
by the senior judges of the Supreme Court. motive. Other factors could also be involved in the
This option is not necessarily true based on the decision.
information provided in the passage. The passage c) The author assumes that the government's decision
does not mention the reaction of senior judges to to give Arjun Ram Meghwal independent charge of
Kiren Rijiju's remarks. the law ministry is solely based on optics.
b) Kiren Rijiju's confrontational rhetoric did not align This assumption is not explicitly made in the passage.
with the government's opinion on judges' While the passage mentions the government's faith in
appointments. optics, it does not provide conclusive evidence that
This option is not supported by the information optics was the sole reason behind the decision.
provided in the passage. The passage suggests that d) The author assumes that Kiren Rijiju's transfer to a
Kiren Rijiju was representing the government's less important ministry will significantly change
opinion on judges' appointments. public perception of the government's attitude
d) Kiren Rijiju's aggressive approach towards the towards the judiciary.
judiciary negatively affected the government's image. This assumption is based on the speculation presented
The passage does not provide evidence to support this in the passage, but it is not explicitly stated as a
inference. It only speculates about the government's guaranteed outcome. The author assumes a cause-
motive for the change in portfolio, but it does not and-effect relationship between the transfer and
explicitly state that Kiren Rijiju's approach negatively public perception without providing conclusive
affected the government's image. evidence.

14. Correct Option A 15. Correct Option B


(H & S) (H & S)
Correct option: a) The author assumes that Kiren Correct option: b) Clearly explain the reasons behind
Rijiju's aggressive remarks were the primary reason Kiren Rijiju's change in portfolio to the public and
for his transfer to the earth sciences ministry. ensure transparency in the decision-making process.
Reasoning: Reasoning:
The passage presents speculation about the The passage highlights the speculation and lack of
government's motive for Kiren Rijiju's change in explanation surrounding Kiren Rijiju's change in
portfolio, suggesting that the transfer may be a portfolio. To address the situation and change public
strategic move to establish a smoother relationship perception, the government should provide clear and

[46]
transparent reasons for the decision. This would help Option B) refers specifically to Robert Koch's
dispel speculation and demonstrate accountability. experiments and does not encompass the broader
Other options: theme discussed in the passage. Option C) focuses on
a) Take immediate steps to repair Kiren Rijiju's the changing view regarding bacteria eradication,
relationship with the judiciary and reinstate him as the which is only a part of the overall main idea. Option
Union law minister. D) touches upon the oversimplification of
This option assumes that the relationship between categorizing bacteria but does not encompass the full
Kiren Rijiju and the judiciary needs to be repaired, scope of the passage.
which is not explicitly mentioned in the passage.
Additionally, reinstating him as the Union law 17. Correct Option C
minister may not address the speculated motive (H & S)
behind his transfer. Correct option: C) The categorization of bacteria as
c) Appoint a new high-profile minister who can parasites, commensals, or mutualists is a valid and
continue Kiren Rijiju's aggressive approach towards accurate representation of their interactions with
the collegium system of judges' appointments. hosts.
This option does not address the desire for a smoother Explanation: This option correctly assumes that the
relationship with the Supreme Court mentioned in the categorization of bacteria as parasites, commensals,
passage. or mutualists is a valid representation of their
d) Postpone the Lok Sabha elections to allow for a interactions with hosts. The passage discusses these
longer period of smooth relations with the Supreme categories and acknowledges their existence,
Court. implying that they are valid descriptors.
This option will not to its full extent address the Option A) makes an incorrect assumption about the
situation in the passage. This will only 'postpone' and historical perception of microbes being solely
not 'address'. responsible for the negative view. Option B) assumes
that the discovery of the human microbiome has
PASSAGE IV completely overturned the understanding of microbes,
16. Correct Option A which is not stated in the passage. Option D) assumes
(H & S) that the recognition of the beneficial role of microbes
Correct option: A) Microbes were initially viewed as has led to a disregard for the potential harm caused by
agents of disease and destruction, but recent certain bacteria, which is not supported by the
discoveries have revealed their essential role in passage.
human health and well-being.
Explanation: This option accurately captures the main 18. Correct Option C
idea of the passage. It highlights the shift in (H & S)
perspective regarding microbes from being seen as Correct option: C) The variability in the behavior of
harmful agents to being recognized as essential for bacteria suggests that their impact on the host is
human health. The passage discusses the historical influenced by factors such as strain and host species.
perception of microbes, the discovery of the human Explanation: This option can be inferred from the
microbiome, and the understanding that not all passage, which discusses the variability of bacteria's
microbes are harmful. behavior and their ability to slide along a spectrum of
interactions with hosts. The passage also mentions the

[47]
influence of factors such as strain and host species on with hosts, suggesting that the categorization is an
the behavior of bacteria. oversimplification.
Option A) contradicts the passage, which states that Option A) goes beyond the information provided in
most bacteria found in the human body are harmless. the passage, as it states a complete transformation of
Option B) makes a generalization that is not supported understanding that is not explicitly mentioned. Option
by the passage. Option D) goes beyond the B) makes a generalization that is not supported by the
information provided in the passage, as the passage. Option D) assumes that the negative
development of effective treatments is not discussed. perception of bacteria has remained unchanged since
the 19th century, which is not stated in the passage.
19. Correct Option A
(H & S) PASSAGE V
Correct option: A) Recent research has demonstrated 21. Correct Option C
that the elimination of certain strains of bacteria from (H & S)
the human microbiome can lead to an increased risk The correct answer is C. The passage states that
of autoimmune diseases. prejudice can lead to systemic discrimination, which
Explanation: This option strengthens the argument by disproportionately affects certain groups, including
providing evidence of the potential harm caused by those who experience poverty and lack of access to
the elimination of certain bacteria from the human healthcare. The other options are either not mentioned
microbiome. It aligns with the passage's argument in the passage or are not the potential harm of
against the view that all bacteria must be eradicated prejudice.
and supports the idea that bacteria play a beneficial Option A is partially correct as the passage also
role in human health. mentions that experiencing prejudice can have serious
Option B) assumes the existence of a universally mental health costs, including anxiety, depression,
beneficial strain of bacteria, which is not discussed in and PTSD. However, this is not the potential harm of
the passage. Option C) mentions the reduction of prejudice that is explicitly stated in the question.
harmful bacteria through antibiotic use, but it does not Option B is also partially correct as the passage states
directly support the argument presented. Option D) that experiencing prejudice can lead to absenteeism
presents a general statement about bacterial infections and decreased productivity. However, this is not the
but does not specifically address the argument in the potential harm of prejudice that is explicitly stated in
passage. the question.
Option D is incorrect as it discusses contact theory
20. Correct Option C rather than the potential harm of prejudice.
(H & S)
Correct option: C) The categorization of bacteria as 22. Correct Option A
parasites, commensals, or mutualists is a flawed (H & S)
oversimplification of their complex nature. The correct answer is A. The passage suggests that
Explanation: This statement can be concluded based cognitive retraining involves exposing people to
on the passage's argument against the categorization counter-stereotypical information to challenge their
of bacteria as solely "good" or "bad." The passage biases. The assumption underlying this suggestion is
emphasizes the continuum of lifestyles that bacteria that people can be persuaded to change their biases
exist along and the variable nature of their interactions through exposure to such information.

[48]
The passage does not suggest that the prevalence of (H & S)
prejudice in society is too great to overcome. Instead, The correct answer is B. The passage states that being
it offers several interventions that have been proven subjected to racism increases the risks of anxiety,
to reduce prejudice, including cognitive retraining depression, post-traumatic stress disorder, and other
and perspective-taking. Therefore, option B is psychological disorders. This can have adverse
incorrect. effects on productivity because poor mental health
C) While the passage does suggest that a lack of leads to absenteeism. Therefore, option B can be
personal experience with other social groups can inferred from the passage.
contribute to prejudice, it does not suggest that this is Option A is incorrect because it only suggests a
the primary cause of prejudice. Instead, it suggests potential consequence of prejudice on work
that prejudice can stem from a variety of factors, attendance, but it doesn't address the mental health
including media portrayals and the beliefs of family costs of prejudice.
and peers. Therefore, option C is incorrect. Option C is incorrect because it suggests that
D) While the passage does suggest that a lack of prejudice only affects those who experience poverty
personal experience with other social groups can and lack of access to healthcare, which is not
contribute to prejudice, it does not suggest that this is necessarily true. Prejudice can affect individuals from
the primary cause of prejudice. Instead, it suggests all socioeconomic backgrounds.
that prejudice can stem from a variety of factors, Option D is partially correct, as prejudice can have
including media portrayals and the beliefs of family severe detrimental effects on individuals and broader
and peers. Therefore, option D is incorrect. society. However, it doesn't specifically address the
mental health costs of prejudice.
23. Correct Option A
(H & S) 25. Correct Option C
The correct answer is A. The passage suggests that (H & S)
contact theory may not be the most effective way to The correct answer is C. While the passage suggests
reduce prejudice because contact alone does not that perspective-taking can be an effective
necessarily lead to reduced prejudice, and the quality intervention for reducing prejudice, it also mentions
and nature of the contact matters as well. Option A that it may not be effective for people who already
strengthens this suggestion by pointing out that hold deeply ingrained biases. Therefore, option C is a
contact that is perceived as forced or insincere may potential limitation of perspective-taking as an
actually reinforce existing prejudices. intervention for reducing prejudice.
Option B is incorrect because it is already mentioned Option A is incorrect because it is actually the
in the passage that the quality and nature of the suggested benefit of perspective-taking in the
contact matter. Option C is incorrect because it passage, rather than a potential limitation. The
contradicts the suggestion made in the passage. passage does not provide evidence to support option
Option D is also incorrect because it is already D, which suggests that perspective-taking has been
mentioned in the passage and doesn't provide any proven to work in the real world. Option D may be too
additional information that would strengthen the broad and not specific enough to the context of the
suggestion made in the passage. passage. Option B may be a potential limitation of
perspective-taking, but the passage does not provide
24. Correct Option B information to support this claim.

[49]
Section III primarily belong to the Mesolithic period, providing
PASSAGE I valuable insights into the ancient human civilization.
1. Correct Option B Hence, the correct option is C.
(H & S)
Answer: B) True fresco paintings are created when the 5. Correct Option C
surface of the wall is dry, allowing the colors to (H & S)
penetrate deeply. Answer: C) G. Kishan Reddy
The surface of the walls should be wet to create a true Solution: G. Kishan Reddy is currently serving as
fresco painting so that the colors will be able to Minister of Tourism & Culture. He belongs to the
penetrate deeply. Madhya Pradesh constituency.
Hence, the correct option is B. Hence, the correct answer is C.

2. Correct Option C PASSAGE II


(H & S) 6. Correct Option C
C) Suresh K. Nair (H & S)
Suresh K. Nair is currently working as an assistant Answer: C) Both 1 and 2
professor of painting at the Department of Painting, Solution: Both the given statements are true.
Faculty of Visual Arts. He and his students initiated Hence, the correct option is C.
the Wall of Peace project in January 2019 in
Cherpulassery, Kerala. 7. Correct Option B
Hence, the correct option is C. (H & S)
Answer: B) KS Radhakrishnan
3. Correct Option B Solution: Supreme Court had set up the three-member
(H & S) LS Radhakrishnan panel on road safety in April 2014.
Answer: B) Ellora caves Hence, the correct option is B.
The Gupta period is considered as the golden age of
art and architecture in India. The Gupta rulers were 8. Correct Option C
great patrons of arts, literature, and scholars. The (H & S)
Ajanta and Ellora caves were carved during the reign Answer: C) Implementing automated speed detection
of the Gupta Empire. They contain paintings of both systems
Hindu and Buddhist themes. Solution: While all the options listed can contribute to
Hence, the correct answer is B. reducing road accidents in India, option C has proven
to be an effective measure. These systems use
4. Correct Option C technology to monitor vehicle speeds and
(H & S) automatically detect and penalize speeding offenders.
Answer: C) Mesolithic period Hence, the correct option is C.
The Bhimbetka rock shelters are a UNESCO World
Heritage Site located in the state of Madhya Pradesh, 9. Correct Option B
India. These rock shelters contain one of the oldest (H & S)
known prehistoric art sites in South Asia, which Answer: B) Section 161(1)(b)

[50]
Solution:161(1)(b) - “hit and run motor accident” Answer: d. All of the above
means an accident arising out of the use of a motor Explanation: The GANHRI cited reasons such as lack
vehicle or motor vehicles the identity whereof cannot of diversity in staff and leadership, insufficient action
be ascertained in spite of reasonable efforts for the to protect marginalized groups, involving the police
purpose. in probes into human rights violations, and poor
Hence, the correct option is B. cooperation with civil society.

10. Correct Option D 13. Correct Option D


(H & S) (H & S)
Answer: D) None of the above Answer: D. Mandate and competence, Autonomy
Solution: Statement 1 is incorrect. It was set up in from government, Independence guaranteed by a
1986 and not in 1995. statute or Constitution, Pluralism, Adequate
Statement 2 is incorrect. Vivek Gogia is the current resources, Adequate powers of investigation
Director of the NCRB. Explanation: The Paris Principles set out six main
Statement 3 is incorrect. It works under the Ministry criteria that NHRIs are required to meet: mandate and
of Home Affairs. competence, autonomy from government,
Hence, the correct option is D. independence guaranteed by a statute or Constitution,
pluralism, adequate resources, and adequate powers
PASSAGE III of investigation.
11. Correct Option A
(H & S) 14. Correct Option A
Answer: a. International benchmarks against which (H & S)
NHRIs can be accredited Answer: a. The highest rating given by the GANHRI
Explanation: The Paris Principles are a set of to NHRIs that follow the Paris Principles
international benchmarks against which National Explanation: ‘A’ status is the highest rating given by
Human Rights Institutions (NHRIs) can be the GANHRI to National Human Rights Institutions
accredited. They provide guidelines for the (NHRIs) that follow the Paris Principles. It allows
establishment, independence, and effectiveness of them to join the work of the GANHRI and the UN on
NHRIs. human rights issues. The NHRC got its ‘A’ rating in
Option b is incorrect because there is no mention of 1999 and kept it in 2006, 2011, and 2017 after a delay
guidelines for conducting investigations into human
rights violations in the text. Option c is incorrect 15. Correct Option A
because the Paris Principles are not a UN resolution (H & S)
on the protection of human rights defenders. Option d Answer: a. International benchmarks against which
is incorrect because the Paris Principles do not NHRIs can be accredited
provide criteria for selecting members of national Explanation: The Paris Principles are a set of
human rights commissions. Hence the correct option international benchmarks against which National
is A Human Rights Institutions (NHRIs) can be
accredited. They provide guidelines for the
12. Correct Option D establishment, independence, and effectiveness of
(H & S) NHRIs.

[51]
Option b is incorrect because there is no mention of Answer: B) They adversely affect the station heat rate
guidelines for conducting investigations into human of plants
rights violations in the text. Option c is incorrect Explanation: Continuing to operate generating units
because the Paris Principles are not a UN resolution that are more than 25 years old is not a bad idea since
on the protection of human rights defenders. Option d the station heat rate of well-maintained plants does
is incorrect because the Paris Principles do not not get adversely affected with age. Option A is
provide criteria for selecting members of national incorrect as it does not accurately reflect the main
human rights commissions. disadvantage of continuing to operate generating units
that are more than 25 years old. Option C is incorrect
PASSAGE IV as it is not relevant to the question. Option D is
16. Correct Option C incorrect as it contradicts the statement in the text that
(H & S) transmission links and coal linkages are already there.
Answer: C) To align with the global trend of phasing Hence the correct option is B
out coal
Explanation: The move to stop building new coal- 20. Correct Option D
fired power plants aligns with the global trend of (H & S)
phasing out coal and shifting to cleaner sources of Answer: D) Enhancing grid reliability and resilience
energy. Options A, B, and D are incorrect as they do Explanation: Diversifying India's energy sources can
not accurately reflect the main reason for India's reduce its reliance on coal and enhance grid reliability
decision. Hence the correct option is C and resilience. Options A, B, and C are incorrect as
they do not accurately reflect the main advantage of
17. Correct Option B diversifying India's energy sources. Hence the correct
(H & S) option is D
Answer: B) 500 GW
Explanation: India plans to achieve an installed PASSAGE V
renewable energy capacity of 500 GW by 2030. 21. Correct Option A
Options A, C, and D are incorrect as they do not (H & S)
accurately reflect India's proposed renewable energy The Kudankulam Nuclear Power Plant in Tamil Nadu
capacity target. Hence the correct option is B is a joint venture project.Russia has offered a more
advanced fuel option for the Kudankulam Nuclear
18. Correct Option C Power Plant (KKNPP) in Tamil Nadu.
(H & S) Hence, Option A is the correct answer.
Answer: C) 60%
Explanation: Nearly 60% of total electricity generated 22. Correct Option C
in India is from coal. Options A, B, and D are (H & S)
incorrect as they do not accurately reflect the The Atomic Energy Regulatory Board (AERB) is the
percentage of total electricity generated in India that regulatory body responsible for overseeing the safety
comes from coal. Hence the correct option is C aspects of nuclear power plants in India, including
those operated by NPCIL. The mission of the Board
19. Correct Option B is to ensure that the use of radiation and Nuclear
(H & S)

[52]
energyn India does not cause undue risk to health and opinions of the government should survive, as bad
the environment opinions were detrimental to the government and
Hence, Option C is the correct answer. monarchy. The law was originally drafted in 1837 by
Thomas Macaulay, the British historian-politician,
23. Correct Option D but was inexplicably omitted when the IPC was
(H & S) enacted in 1860.
NTPC Limited, formerly known as National Thermal
Power Corporation, is an Indian central Public Sector 28. Correct Option B
Undertaking under the ownership of the Ministry of (H & S)
Power, Government of India which is engaged in the B-The first case to be tried under the law of sedition
generation of electricity and allied activities. in India was 20 years after its introduction in Queen-
Hence, Option D is the correct answer. Empress v Jogendra Chunder Bose, 1891.

24. Correct Option B 29. Correct Option A


(H & S) (H & S)
Air India is not a Central Public Sector Undertaking A- Sedition was devised as a tool in 13th century
(PSU). It is currently owned by the Tata Group after Britain to suppress the freedom of the printing press
the Government of India disinvested it completed in and its ability to criticise the King. The Sedition Act,
2021. 1661 imposed punishment on anyone who wrote,
Hence, Option B is the correct answer. printed or preached any words against the King

25. Correct Option B 30. Correct Option C


(H & S) (H & S)
Jitendra Singh is the Minister of State for Atomic C- As early as 1950, the SC in Romesh Thapar v State
Energy and Space. The ministry is headed by the of Madras held that “criticism of the government
Prime Minister. exciting disaffection or bad feelings towards it, is not
Hence, Option B is the correct answer. to be regarded as a justifying ground for restricting the
freedom of expression and of the press, unless it is
PASSAGE VI such as to undermine the security of or tend to
26. Correct Option A overthrow the state.”
(H & S)
According to IPC Section 124 A, “Whoever brings or Section IV
attempts to bring into hatred or contempt, or excites PASSAGE I
or attempts to excite disaffection towards the 1. Correct Option D
Government established by law in India shall be (H & S)
punished. answer- d) No, since suresh is a Man and the
intercoure should be between person of opposite sex
27. Correct Option C Adultery, as defined in the scenario, involves
(H & S) consensual and voluntary intercourse between a
C- Sedition laws were enacted in 17th century married person and another person of the opposite
England when lawmakers believed that only good sex. In this case, as Suresh is also a man, Paro would

[53]
not be able to file for divorce on the ground of To strengthen Hamina's case, she needs to establish
adultery based on the scenario provided. Adultery, as two key elements: (A) that intercourse between Rohit
legally recognized, requires a sexual relationship and Reshma was consensual, and (B) at the time of
between a married person and a person of the opposite the act, the marriage between Reshma and Suresh was
sex subsisting. Both these elements are essential to prove
Hence, the correct option is (D) adultery under the Hindu Marriage Act and strengthen
the grounds for divorce
2. Correct Option C Hence, the correct option is (D)
(H & S)
Answer- c) Yes, since at that time marriage was 5. Correct Option D
subsisting, Maegan had intercousre with the opposite (H & S)
sex and adultery is considered as one of the most Answer- d) No, since abusive behavior towards the
important ground for seeking divorce assistant does not directly prove that he will do same
according to the passage, with his wife and cruelty means when one spouse
One of the spouses involved in the intercourse with beats or causes any bodily injury to the other
another person, married or unmarried, of the opposite The grounds for filing divorce on the basis of cruelty
sex. require abusive behavior towards the spouse. While
Intercourse should be voluntary and consensual. Pranil's behavior towards his assistant is undoubtedly
At the time of the act, the marriage was subsisting. abusive, it does not directly prove that he will exhibit
There must be sufficient circumstantial evidence to the same behavior towards his wife. Furthermore, the
prove the liability of another spouse. definition of cruelty in this context refers to physical
all criteria of adultery are fulfilled here so Paul is harm or injury inflicted by one spouse upon the other,
likely to succeed which does not match the scenario described
Hence, the correct option is (C) Hence, the correct option is (D)

3. Correct Option A PASSAGE II


(H & S) 6. Correct Option D
Answer- a) Yes, since lafik left her without any (H & S)
reasonable justification and without her consent The RTI Act mandates that public authorities provide
Desertion refers to the permanent abandonment of one access to information held by them, unless it falls
spouse by the other without any reasonable within the specified exemptions. The economic
justification and without their consent and according interests of the country are not explicitly mentioned
to the question he left her because he considers her as an exemption, and therefore, the PIO's decision to
unlucky which is not a resonable justification to deny the request based on economic interests is not
abandon his wife without her consent. justified. Therefore the correct answer is d a) and b)
Hence, the correct option is (A) are incorrect because the economic interests of the
country do not automatically override the right to
4. Correct Option D access information. The PIO must disclose
(H & S) information unless it falls within the specified
Answer- d) both a and b exemptions.

[54]
c) is incorrect because it falsely implies that the 9. Correct Option A
citizen's right to access information can be denied (H & S)
based on economic interests, which is not supported The correct answer is a) Yes, because the citizen has
by the RTI Act.Hence the correct answer is d the right to appeal if the public authority fails to
respond within the specified timeframe.
7. Correct Option A Section 5(2) of the RTI Act provides for the
(H & S) designation of Assistant PIOs who receive
The correct answer is a) Yes, because the disclosure applications and appeals. If a citizen does not receive
of information about ongoing investigations can a response from the PIO within the specified
hinder the investigative process. timeframe, they have the right to file an appeal.
The RTI Act includes exemptions to the disclosure of b) is incorrect because it incorrectly suggests that the
information, and one of the exemptions is for ongoing citizen can seek remedies for non-compliance with
investigations. Disclosing information about ongoing the Act, but the primary remedy in this case is the
investigations can potentially interfere with the right to appeal.
investigative process, and therefore, the denial of the c) is incorrect because it falsely states that the RTI Act
request based on this ground is justified. does not provide for the right to appeal in case of non-
b) is incorrect because it suggests that protecting the response.
integrity of ongoing investigations takes precedence d) is incorrect because it wrongly assumes that the
over the right to information, which is not explicitly specified timeframe is not legally binding.
stated in the RTI Act.
c) is incorrect because it disregards the exemption for 10. Correct Option C
ongoing investigations mentioned in the RTI Act. (H & S)
d) is incorrect because the RTI Act provides The correct answer is c) No, because the RTI Act
exemptions for ongoing investigations. mandates public authorities to disseminate
information in an easily accessible manner.
8. Correct Option B Section 4(3) of the RTI Act emphasizes the wide
(H & S) dissemination of information in an easily accessible
The correct answer is b) Yes, because the citizen has manner. While cost-effectiveness and effectiveness
the right to hold the public authority accountable for are important considerations, the public authority
non-compliance with the RTI Act. cannot refuse to provide information solely based on
Section 4(1) of the RTI Act mandates that public the lack of resources for dissemination. a) and b) are
authorities publish certain particulars within 120 days incorrect because they wrongly suggest that the lack
of the Act's enactment. If a public authority fails to of resources can justify the refusal to disseminate
comply with this obligation, a citizen can file a information, which is not supported by the RTI Act.
complaint to ensure accountability. d) is incorrect because it assumes that the public
a) is incorrect because it only mentions the legal authority's lack of resources should
obligation of the public authority without considering
the citizen's right to hold them accountable. PASSAGE III
c) and d) are incorrect because they downplay the 11. Correct Option B
significance of holding public authorities accountable (H & S)
for non-compliance with the RTI Act.

[55]
Answer- b) Uttam is liable under section 375 as he Hence, the correct option is (D)
applied his mouth on her vagina, which comes under
rape. 14. Correct Option C
Reason: According to the given scenario, Uttam (H & S)
entered Raji's room and applied his mouth on her Answer- c) Susmita should file case Under section
vagina. This act falls within the definition of rape 376 also her consent was not vaild as she gave her
under Section 375 of IPC, which includes the consent under threat and maximum punishment is life
application of mouth into the vagina, urethra, or anus Imprisonment
of a woman. It is not necessary for penetration to Reason- The reason Susmita should file a case under
occur for an act to be considered rape. In this case, section 376 is because the scenario described involves
Uttam's action of applying his mouth on Raji's vagina the use of threat and coercion, as Roman, Pinku, and
constitutes rape, irrespective of whether penetration Dinku showed Susmita a gun and asked her for sexual
took place or not. Therefore, Uttam is liable under activity. In such a situation, even if Susmita initially
section 375 agrees due to fear or coercion, her consent is not
Hence, the correct option is (B) considered valid under the law. Section 376 typically
deals with cases of rape where consent is obtained
12. Correct Option D under duress or threat. The maximum punishment
(H & S) under section 376 is life Imprisonment
Answer- d) Raja is not liable because Rohan is not a Hence, the correct option is (C)
woman, and rape can only be committed against a
woman. 15. Correct Option B
Reason: In this scenario, it is mentioned that Raja, (H & S)
who is intoxicated, mistakenly enters Rohan's room Answer- b) Susmita should not file any case against
and puts his finger inside Rohan's anus, believing them as it was consensual, and there was no
Rohan to be his wife. However, it is important to note apprehension of threat.
that rape, as defined under Section 375 of IPC, can Reason- The reason for this choice is that as described
only be committed against a woman. Since Rohan is in the scenario tis was consensual act without any
a biological male, even if he behaves like a woman, indication of threat or coercion. Consent plays a
the act cannot be classified as rape under the given crucial role in determining the legality of a sexual act,
circumstances. Therefore, Raja would not be held and in this situation, Susmita willingly agreed without
liable under section 375 for rape in this case any apparent coercion. Therefore, filing a case under
Hence, the correct option is (D) section 376, which involves non-consensual acts or
acts involving coercion, would not be appropriate
13. Correct Option D Hence, the correct option is (B)
(H & S)
Answer- d) Vladimir is not liable as she is above the PASSAGE IV
age of 18 and her consent is valid 16. Correct Option B
Reason- In this case Vladimir and Natalia had sex on (H & S)
3rd january 2024 and on that day she was more than Answer: b) This law is valid as sacrificing children is
18 so her consent is vaild. consent can be written,oral an immoral act and Article 25 is subject to morality.
or can be shown with action Reason: The law prohibiting the practice of throwing

[56]
children into an open fire as a religious ceremony is managed by the community, their right to religious
valid because it aims to protect the fundamental rights instruction, as protected under Article 28, is violated
and well-being of children. Article 25 guarantees the Hence, the correct option is (D)
freedom of religion but is subject to public order,
health, and morality. In this case, the act of sacrificing 19. Correct Option B
children is considered an immoral act and goes (H & S)
against the principles of public morality and the rights Answer: b) This law is valid as religious freedoms
of the child, outweighing the right to freedom of under Article 25 are subject to morality, and this
religion practice is immoral.
Hence, the correct option is (B) Reason: The law prohibiting the drinking of human
blood as a religious practice is valid as religious
17. Correct Option B freedoms under Article 25 are subject to morality.
(H & S) While Article 25 guarantees the freedom of
Answer: b) This law is valid as consuming drugs is a conscience, profession, practice, and propagation of
health concern and Article 25 is subject to public religion, it is also subject to public order, health, and
health. morality. In this case, the act of drinking human blood
Reason: The law prohibiting the consumption of is considered immoral, and restrictions can be
drugs as part of a religious ceremony is valid because imposed on such practices in the interest of public
it addresses health concerns. Article 25 guarantees the morality
freedom of religion but is subject to public order, Hence, the correct option is (B)
health, and morality. In this case, the consumption of
drugs poses health risks, and the state has a legitimate 20. Correct Option A
interest in protecting public health and preventing (H & S)
potential harm associated with drug use, which Answer: a) There is a violation of Article 26 (Freedom
outweighs the right to freedom of religion to manage religious affairs), which includes the
Hence, the correct option is (B) purchase of immovable property.
Reason: There is a violation of Article 26 in this case.
18. Correct Option D Article 26 guarantees the freedom to manage religious
(H & S) affairs, including the right to form and maintain
Answer: d) religious rights under Article 28 are institutions for religious and charitable purposes. The
violated here as it permits educational institutions that Pontie community's right to purchase immovable
are maintained by religious groups to disseminate property is an aspect of managing their religious
religious instruction. affairs. By preventing them from purchasing the
Reason: The right violated in this case is the religious property, their right under Article 26 is violated
rights under Article 28. Article 28 guarantees freedom Hence, the correct option is (A)
as to attendance at religious instruction or religious
worship in certain educational institutions. It permits PASSAGE V
educational institutions maintained by religious 21. Correct Option D
groups to disseminate religious instruction. By (H & S)
prohibiting the PCN community from disseminating Answer: d) Bigamy
religious knowledge in educational institutions

[57]
Reason: Under Section 494 of the Indian Penal Code, if the spouse of anyone is not dead at the time of
the offense of bigamy is punishable. Bigamy refers to marriage, then that would be considered as the offense
the act of marrying someone while already being of bigamy.
legally married to another person Reason: According to Section 5(i) of the Hindu
Hence, the correct option is (D) Marriage Act, only monogamy is accepted in the
Hindu religion. If one's spouse is still alive at the time
22. Correct Option A of marriage to another person, it would be considered
(H & S) as the offense of bigamy. Shizuka's claim of bigamy
Answer: a) No, they have not reached the legal age of is based on this provision of the Hindu Marriage Act
marriage as the legal age for marriage for boys is 21. Hence, the correct option is (A)
Reason: According to the Hindu Marriage Act, the
legal age for marriage is 21 years for males. As both PASSAGE VI
Jungkook and Rashi are 18 years old, they have not 26. Correct Option A
reached the legal age of marriage under the Act (H & S)
Hence, the correct option is (A) Correct answer: a) Yes, because the RTI Act includes
civil societies indirectly funded by public funds as
23. Correct Option D public authorities.
(H & S) Explanation: Section 2(h) of the RTI Act defines
Answer: d) both a and b public authority as any authority or body under the
Reason: Section 5(v) of the Hindu Marriage Act Constitution of India, including civil societies
prohibits marriages with individuals who are within indirectly or directly funded by public funds.
the prohibited degrees of relationship, including Therefore, a private company receiving funding from
marriages with the third generation in the line of the central government for a specific project is
ascent through the mother and the fifth generation in considered a public authority under the RTI Act.
the line of ascent through the father b) Yes, because the private company is carrying out a
Hence, the correct option is (D) project under the monitoring of the central
government: This option is incorrect because the
24. Correct Option D monitoring of the central government does not solely
(H & S) determine the classification of a private company as a
Answer: d) all the above public authority under the RTI Act. The key criterion
Reason: Marriage serves multiple purposes, including is the indirect or direct funding from public funds.
the social practice of uniting two individuals and their c) No, because the RTI Act only applies to state and
families, granting them the rights to conjugal central government monitored institutions: This
relations, and providing a socially approved way of option is incorrect because Section 2(h) of the RTI
establishing a family of procreation Act explicitly includes civil societies indirectly or
Hence, the correct option is (D) directly funded by public funds as public authorities,
irrespective of whether they are state or central
25. Correct Option A government monitored institutions.
(H & S) d) No, because private companies are exempt from
Answer: a) Sec. 5(i) of the Hindu Marriage Act says disclosing information under Section 4 of the RTI
that only monogamy is accepted in Hindu religion and Act: This option is incorrect because Section 4 of the

[58]
RTI Act pertains to the proactive disclosure of information related to ongoing investigations under
information by public authorities. It does not imply an the RTI Act.
exemption for private companies funded by public a) Yes, because Section 2(j) of the RTI Act grants
funds. access to any information under the control of a public
authority: This option is incorrect because although
27. Correct Option C Section 2(j) provides the definition of "Right to
(H & S) Information," it does not override the exemptions
Section 1(2) of the RTI Act clearly states that the Act mentioned in Section 8, which include ongoing
extends to the whole nation except for the State of investigations.
Jammu and Kashmir. Therefore, citizens of Jammu b) Yes, because the citizen has a right to know about
and Kashmir are not entitled to exercise their right to the actions of public authorities: This option is
information under the RTI Act.a) Yes, because the incorrect because while citizens have the right to
RTI Act extends to the whole nation, including information, the exemptions specified in Section 8 of
Jammu and Kashmir: This option is incorrect because the RTI Act limit the scope of information that can be
it contradicts the information provided in Section 1(2) disclosed.
of the RTI Act, which specifically excludes Jammu d) No, because public authorities are not required to
and Kashmir.b) Yes, because the right to information disclose information under Section 6 of the RTI Act:
is a fundamental right of speech and expression: This This option is incorrect because Section 6 of the RTI
option is incorrect because although the RTI Act Act outlines the procedure for securing information,
enables citizens to exercise their fundamental right to but it does not pertain to exemptions from disclosure.
speech and expression, it does not apply to Jammu
and Kashmir as per Section 1(2).d) No, because the 29. Correct Option B
RTI Act does not apply to civil societies indirectly (H & S)
funded by public funds: This option is incorrect Correct answer: b) Yes, because Section 20 of the RTI
because it is unrelated to the question at hand. The Act provides penalty fees for delayed or distorted
exemption of Jammu and Kashmir from the RTI Act information.
is not based on civil societies indirectly funded by Explanation: Section 20 of the RTI Act provides for
public funds. the imposition of penalties if there is a delay in
Hence, option C is correct providing information or if the information provided
is distorted or wrong. Therefore, penalty fees can be
28. Correct Option C imposed on the PIO for the delay in providing
(H & S) information.
Correct answer: c) No, because ongoing a) Yes, because Section 7 of the RTI Act prescribes a
investigations fall under the exemption clause time frame of 30 days for providing information: This
mentioned in Section 8 of the RTI Act. option is incorrect because Section 7 of the RTI Act
Explanation: Section 8 of the RTI Act provides for does prescribe a time frame for providing information
exemptions from disclosing certain categories of but does not directly address the imposition of penalty
information, including information that would impede fees.
the process of an ongoing investigation. Therefore, c) No, because the RTI Act does not impose any
public authorities are not obligated to disclose penalties for delayed responses by the PIO: This
option is incorrect because Section 20 of the RTI Act

[59]
explicitly provides for the imposition of penalty fees 1. Correct Option B
for delays and other specific situations. (H & S)
d) No, because penalty fees can only be imposed if the From the question:
information provided is wrong or distorted: This (𝑥 + 200) × 12 × 2 𝑥 × 10 × 2
100
− 100
= 240
option is incorrect because penalty fees can be 24𝑥 + 4800 20𝑥
100
− 100
= 240
imposed for both delayed information and for
4𝑥 + 4800
distorted or wrong information, as mentioned in 100
= 240
Section 20 of the RTI Act. x = 4800
Now, the SI received on 4800 - 600 = ₹4200 at 15%
30. Correct Option C rate after 3 years:
(H & S) 4200 × 15 × 3
100
= ₹1890
Correct answer: c) No, because Section 23 of the RTI
Act bars the lower courts from entertaining such suits
2. Correct Option D
or applications.
(H & S)
Explanation: Section 23 of the RTI Act states that the
If the number 13P7Q is divisible by 11.
lower courts are barred from entertaining any suits or
So,
applications regarding the Act. However, the writ
(1 + P + Q) - (3 + 7) = 0 or 11
jurisdiction of the Supreme Court and High Courts
P + Q = 9 or P + Q = 20
remains intact under Article 32 and 225 of the
Case I: If, P + Q = 9
Constitution. Therefore, lower courts cannot entertain
And, P - Q = 1
suits or applications related to the RTI Act.
So, P = 5 and Q = 4
a) Yes, because the lower courts have jurisdiction
Case II: If, P + Q = 20
over matters related to the RTI Act: This option is
And P - Q = 1
incorrect because Section 23 explicitly bars the lower
So, P = 10.5 and Q = 9.5
courts from entertaining suits or applications
Case II can be neglected.
regarding the RTI Act.
So, the number is 13574.
b) Yes, because the citizen has a right to seek justice
After dividing the number by 9, it gives the remainder
for the denial of information: This option is incorrect
2.
because while citizens have the right to seek justice,
So, we should subtract 2 from the number to make it
Section 23 restricts the jurisdiction of lower courts
divisible by 9.
specifically for matters related to the RTI Act.
d) No, because the writ jurisdiction of the Supreme
3. Correct Option D
Court and High Courts is not affected by Section 23
(H & S)
of the RTI Act: This option is incorrect because
Let age of A be ‘x’ years
Section 23 does not affect the writ jurisdiction of the
And age of B = x+5 years
Supreme Court and High Courts. It only pertains to
According to the question,
the jurisdiction of the lower courts in RTI Act-related 𝑥+4 6
𝑥+5−5
=5
matters.
5x+ 20 = 6x
x= 20
Section V
4
PASSAGE I So, the age of C = 5 × 20 = 16 years

[60]
20+16 3
So, required average = 2
= 18 years Water in 16 liters = 16× 8 = 6 liters

After adding 12 liters of milk


4. Correct Option C Milk quantity = 25 – 10 + 12 = 27 liters
(H & S)
Quantity of Water = 15 – 6 = 9 liters
Let the incomes of Geeta and Sita be 5x and 4x
Ratio of milk and water = 27:9 = 3:1
respectively
3
And the expenditures of Geeta and Sita be 7y and 3y Milk in 20 liters = 20 × 4 = 15 liters
respectively. 1
Water in 20 liters = 20× 4 = 5 liters
ATQ,
Final milk quantity = 27 – 15 + 8 = 20 liters
(5x-7y) = 800 + (4x-3y)
i.e. x = 800 + 4y. …….(1) Water quantity = 9 – 5 = 4 liters
4
Also, 7y = Rs 1400 Required percentage = 20 ×100 = 20%
y = Rs 200.
From (1), x = 800 + 4(200) = 1600. 7. Correct Option C
So, the total income of Geeta and Sita = (5x+4x)= (H & S)
9×1600=14400. (36 - 24)% of total marks = 2x + x
12% of total marks = 3x
5. Correct Option C Total marks = 300x/12 = 25x
(H & S) Kumar's marks = 40% of 25x
Let the distance between A and B is 7x and the 100 = 10x
distance between B and C is 6x. x = 10
Total distance between A and C = 7x + 6x = 13x km Total marks = 250
Total Distance travelled between Pass mark = 24% of 250 + 20 = 60 + 20 = 80 marks
Speed 
Time taken
(7x  6x)
 60  8. Correct Option A
  10  
 2   60   (H & S)
  
Let cost price of 1 kg = 100
130
 13x  60  Marked price of 1 kg = 120% of 100 = 120
60
Selling price of 1 kg when uses true weight
⇒ x = 10
= 75% of 120 = 90
Therefore, the distance from A to
Selling price of 1 kg when uses false weight
B = 7x = 7 × 10 = 70km.
1000
= 90 × ( 750 ) = 120
So, the answer is that the distance from A to B is
70 km Profit amount = 120 – 100 = 20
20
Profit percent = (100) × 100 = 20%
6. Correct Option C
(H & S)
5
Milk in 40 liters = 8 × 40= 25 liters
3
Water in 40 liters = 8 × 40 = 15 liters
5
Milk in 16 liters = 16 × = 10 liters
8

[61]
9. Correct Option A 10. Correct Option A
(H & S) (H & S)
P and Q together complete the work in = 32 × 2 = 64 Let the speed of the swimmer = x km/h
36 And the speed of the stream = y km/h
days Q and R together can complete the work in = 60
Now,
× 100 = 60 days
84
Let the total work = 960 units 𝑥+𝑦
=6
960
Efficiency of (P + Q) = = 15 units/day x + y = 14………(i)
64
36
960
𝑥−𝑦
=6
Efficiency of (Q + R) = = 16 units/day
60
x – y = 6…………(ii)
Now,
Adding eq(i) and eq(ii) we get,
16 × P + 52 × Q + 52 × R = 960
2x = 20
16P = 960 – 52 × 16 ⇒ x = 10
Efficiency of P = 8 units/day From (i)
Efficiency of Q = 15 – 8 = 7 units/day 10 + y = 14
Efficiency of R = 16 – 7 = 9 units/day ⇒ y = 4 km/h
960
Required time = 7 = 60 days
9+8×
8

For more questions, kindly visit the library section: Link for
app: https://links.physicswallah.live/vyJw

For more questions, kindly visit the library section: Link for [62]

You might also like